NCLEX style questions

Réussis tes devoirs et examens dès maintenant avec Quizwiz!

The nurse discusses the recommended weight gain during pregnancy with a newly pregnant client who is 5 ft 3 in (160 centimeters) tall and weighs 130 lb (57 kilograms). The nurse explains that with the recommended weight gain, at term the client should weigh how much? 1 150 lb (68 kg) 2 140 lb (63.5 kg) 3 135 lb (61 kg) 4 130 lb (57 kg)

1 A weight of 150 lb (68 kg) would put the client within the recommended weight gain of at least 25 lb (11 kg) for a woman who was of average weight for her height before pregnancy. A weight of 140 lb (63.5 kg) is less than the recommended weight gain for a woman of average weight for height before pregnancy, as are 135 lb (61 kg) and 130 lb (57 kg). Recommendations are that women with a normal body mass index (BMI) should gain 25 to 35 lb (11.3 to 15.9 kg) during pregnancy.

What is the most important nursing action when measuring a client's pulmonary capillary wedge pressure (PCWP)? 1 Deflate the balloon as soon as the PCWP is measured. 2 Have the client bear down when measuring the PCWP. 3 Place the client in a supine position before measuring the PCWP. 4 Flush the catheter with a heparin solution after the PCWP is determined

1 Although the balloon must be inflated to measure the capillary wedge pressure, leaving the balloon inflated will interfere with blood flow to the lung. Bearing down will increase intrathoracic pressure and alter the reading. Although a supine position is preferred, it is not essential. Agency protocols relative to flushing of unused ports must be followed.

A child has been diagnosed with hemophilia type A after experiencing excessive bleeding from a minor trauma. How is hemophilia inherited? 1 X-linked recessive trait 2 Y-linked recessive trait 3 X-linked dominant trait 4 Y-linked dominant trait

1 Hemophilia A is an X-linked recessive trait, not a dominant trait, meaning daughters who have the gene are carriers, and sons with the gene have the condition. The trait is not carried on the Y chromosome.

A nurse is collecting data from a client with varicose veins who is to have sclerotherapy. What should the nurse expect the client to report? 1 Feeling of heaviness in both legs 2 Intermittent claudication of the legs 3 Calf pain on dorsiflexion of the foot 4 Hematomas of the lower extremities

1 Impaired venous return causes increased pressure, with symptoms of fatigue and heaviness. Pain when walking relieved by rest (intermittent claudication) is a symptom related to hypoxia. Symptoms of hypoxia are related to impaired arterial, rather than venous, circulation. Calf pain on dorsiflexion of the foot is Homans sign, which is suggestive of thrombophlebitis. Ecchymoses may occur in some individuals, but bleeding into tissue is insufficient to cause hematomas.

After a client has a total gastrectomy, the nurse plans to include in the discharge teaching the need for what treatment? 1 Monthly injections of cyanocobalamin 2 Regular daily use of a stool softener 3 Weekly injections of iron dextran 4 Daily replacement therapy of pancreatic enzymes

1 Intrinsic factor is lost with removal of the stomach, and cyanocobalamin is needed to maintain the hemoglobin level once the client is stabilized; injections are given monthly for life. Adequate diet, fluid intake, and exercise should prevent constipation. Weekly injections of iron dextran are not considered routine. Daily replacement therapy of pancreatic enzymes does not affect pancreatic enzymes.

What information does the nurse need to teach a client in order for her to perform an accurate breast self-examination? 1 Squeeze the nipples to examine for discharge. 2 Use the right hand to examine the right breast. 3 Place a pillow under the shoulder opposite the examined breast to raise it. 4 Compress breast tissue to the chest wall with the palm to palpate for lumps.

1 Serous or bloody discharge from the nipple is pathologic and must be reported. The right hand should be used to examine the left breast because this allows the flattened fingers to palpate the entire breast, including the tail (upper, outer quadrant toward the axilla) and axillary area. A small pillow or rolled towel should be placed under the scapula of the side being examined because it helps raise the chest wall and spread and flatten out breast tissue. The flat part of the fingers, not the palm or fingertips, should be used for palpation.

A client is on a cardiac monitor. The monitor begins to alarm showing ventricular tachycardia. What should the nurse do first? 1 Check for a pulse 2 Start cardiac compressions 3 Prepare to defibrillate the client 4 Administer oxygen via an ambu bag

1 The treatment of ventricular tachycardia depends on the presence of a pulse. Therefore checking for a pulse is the first priority for the nurse. The nurse must rely on client assessment, not solely on the monitor. Cardiac compressions would not be initiated if there was a pulse. Administering oxygen via an ambu bag would only occur if the client was not breathing. The client is not automatically defibrillated. Cardioversion is recommended for slower ventricular tachycardia.

Which hormone is crucial for ovulation and complete maturation of a client's ovarian follicles? 1 Luteinizing hormone 2 Follicle stimulating hormone 3 Gonadotropin releasing hormone 4 Human chorionic gonadotropin hormone

1 Ovulation and complete maturation of ovarian follicles can only take place in the presence of luteinizing hormone. However, follicle stimulating hormone initiates maturation of the follicles. Gonadotropin releasing hormone stimulates the pituitary gland to release follicle stimulating hormone and luteinizing hormone. Human chorionic gonadotropin hormone is released after implantation and is responsible for secretion of progesterone and estrogen during pregnancy.

A client in her 37th week of gestation calls the nurse at the clinic and reports, "My ankles are so swollen." Which intervention should the nurse recommend? 1 Limiting fluid intake during the day 2 Elevating her legs more frequently during the day 3 Restricting salt intake for the remainder of her pregnancy 4 Taking a mild diuretic that the healthcare provider will prescribe

2 Dependent edema in the ankles is a common occurrence during the latter part of pregnancy. It results from increased pressure of the uterus on the pelvic veins. Elevating the legs encourages venous return. Limiting fluid intake can be harmful; increased circulating blood volume during pregnancy must be maintained. Salt is necessary to retain fluid for the increased circulating blood volume during pregnancy. Diuretics are not utilized during pregnancy; they may decrease the circulating blood volume.

A nurse is obtaining a health history from a client with newly diagnosed cervical cancer. Which aspect of the client's life is most important for the nurse to explore at this time? 1 Sexual history 2 Support system 3 Obstetric history 4 Elimination patterns

2 During a health crisis the client will need support from significant others. The sexual history is important in diagnosis and the obstetric history and elimination patterns are important parts of the medical history; however, none are the priority at this time.

An infant of a diabetic mother is admitted to the neonatal intensive care unit. What is the priority nursing intervention for this infant? 1 Clamping the cord a second time 2 Obtaining heel blood to test the glucose level 3 Starting an intravenous (IV) infusion of glucose in water 4 Instilling an ophthalmic antibiotic to prevent an eye infection

2 Hypoglycemia may be present because of the sudden withdrawal of maternal glucose and increased fetal insulin production, which continues after birth. The umbilical vein may be needed to start an IV; it should not be damaged. An IV infusion of glucose should not be started until the blood glucose level has been determined. Instilling an antibiotic into the eyes can be delayed until the blood glucose level has been determined.

Before a client has a cardiac catheterization, an electrocardiogram (ECG) is performed, and hypokalemia is suspected. The nurse expects that the diagnosis will be confirmed by which diagnostic test? 1 Complete blood count 2 Serum potassium level 3 X-ray film of long bones 4 Blood cultures times three

2 Hypokalemia is suspected when the T wave on an ECG tracing is depressed or flattened; a serum potassium level less than 3.5 mEq/L (3.5 mmol/L) indicates hypokalemia. Complete blood count, x-ray film of long bones, and blood cultures times three will have no significance in the diagnosis of a potassium deficit.

On the morning of surgery a client is admitted for resection of an abdominal aortic aneurysm. While awaiting surgery, the client suddenly develops symptoms of shock. Which nursing action is priority? 1 Prepare for blood transfusions. 2 Notify the surgeon immediately. 3 Make the client nothing by mouth (NPO). 4 Administer the prescribed preoperative sedative.

2 Immediate surgical intervention to clamp the aorta is necessary for survival; the aneurysm has ruptured. Preparing for blood transfusions may be done eventually, but notifying the surgeon is the priority. The client is already NPO. Sedatives mask important signs and symptoms of shock.

Which term should the nurse use to describe a flat, poorly defined mass on the sole over a bony prominence caused by pressure? 1 Plantar wart 2 Callus 3 Ingrown nail 4 Hypertrophic ungual labium

2 In foot problems, callus is described as a flat, poorly defined mass on the sole over a bony prominence that is caused by pressure. Plantar wart is a painful papillomatous growth caused by a virus. A sliver of toenail penetrating the skin and causing inflammation results in ingrown nail. Hypertrophic ungual labium is described as chronic hypertrophy of the nail lip caused by improper nail trimming.

What is the best advice a nurse can provide to a pregnant woman in her first trimester? 1 "Cut down on drugs, alcohol, and cigarettes." 2 "Avoid drugs and don't smoke or drink alcohol." 3 "Avoid smoking, limit alcohol consumption, and don't take aspirin." 4 "Take only prescription drugs, especially in the second and third trimesters."

2 The first trimester is the period when all major embryonic organs are forming; drugs, alcohol, and tobacco may cause major defects. Cutting down on these substances is insufficient; they are teratogens and should be eliminated. Even 1 oz of an alcoholic drink is considered harmful; baby aspirin may be prescribed to some women who are considered at risk for pregnancy-induced hypertension; however, not during the first trimester. Medications, unless absolutely necessary, should be avoided throughout pregnancy; however, the first trimester is most significant.

During discharge teaching a client who just had a hysterectomy states, "After this surgery, I don't expect to be interested in sex anymore." What should the nurse consider before responding? 1 Many women believe that their sexual function is related to their uterus. 2 Surgically forced menopause usually results in a decreased sex drive. 3 The loss of estrogen that results from this surgery will cause most women to experience a decrease in libido. 4 Body image changes that occur after this surgery prevent many women from resuming sexual activity.

1 The uterus is often erroneously believed necessary for a satisfying sex life. Sexuality after hysterectomy should not be diminished, particularly because the fear of pregnancy no longer exists. Although the estrogen level is reduced, libido is influenced by psychologic as well as hormonal factors. Although body image changes can interfere with sexuality, this is not an expectation for most women.

A client is experiencing an exacerbation of ulcerative colitis. A low-residue, high-protein diet and IV fluids with vitamins have been prescribed. When implementing these prescriptions, which goal is the nurse trying to achieve? 1 Reduce gastric acidity 2 Reduce colonic irritation 3 Reduce intestinal absorption 4 Reduce bowel infection rate

2 A low-residue diet is designed to reduce colonic irritation, motility, and spasticity. Reduction of gastric acidity is the aim of bland diets used in the treatment of gastric ulcers. Reducing colonic irritation, motility, and spasticity hopefully will increase, not reduce, intestinal absorption. This diet is to allow the bowel to rest, not to reduce infection rates.

A 42-year-old client at 39 weeks' gestation has a reactive nonstress test (NST). What should the nurse explain to the client about the positive result? 1 Immediate birth is indicated. 2 This is the desired response at this stage of gestation. 3 Further testing is unnecessary with this desired outcome. 4 The result is inconclusive, indicating the need for further evaluation.

2 An NST indicates that the fetus is healthy because there is an active pattern of fetal heart rate acceleration with movement. The result is positive and desired; immediate birth is not required. Further testing is needed. If the pregnancy continues, another test of fetal well-being will probably be done. The results were positive, not inconclusive.

The nurse is providing postoperative care to a client with lung cancer who had a partial pneumonectomy. When inspecting the client's dressing, the nurse notes puffiness of the tissue around the surgical site. When the nurse palpates the site, the tissue feels spongy and crackles can be felt. How does the nurse describe this assessment finding? 1 Respiratory stridor 2 Subcutaneous emphysema 3 Bilateral 2+ pitting edema 4 Chest distention

2 There is air in the tissues and palpation results in a crackling sound referred to as subcutaneous emphysema. Respiratory stridor is a harsh, high-pitched sound usually produced on inspiration because of airway obstruction. Bilateral 2+ pitting edema is excessive accumulation of fluid in tissue spaces. The size of the chest is determined by the bony structure; a barrel chest with an increase in the anteroposterior (AP) diameter is associated with chronic obstructive pulmonary disease (COPD), not cancer of the lung.

A client is admitted to the hospital with a diagnosis of Crohn disease. What is most important for the nurse to include in the teaching plan for this client? 1 Controlling constipation 2 Meeting nutritional needs 3 Preventing increased weakness 4 Anticipating a sexual alteration

2 To avoid gastrointestinal pain and diarrhea, these clients often refuse to eat and become malnourished. The consumption of a high-calorie, high-protein diet is advised. Diarrhea, not constipation, is a problem with Crohn disease. Preventing an increase in weakness is a secondary concern that results from malnutrition; correcting the malnutrition will increase strength. Anticipating a sexual alteration generally is not a problem with Crohn disease.

Which client is at greatest risk for the development of a venous thrombosis? 1 A 76-year-old female with a 100-pack-per-year smoking history and hypertension 2 A 68-year-old male on bed rest following a left hip fracture 3 A 59-year-old male who is an intravenous drug user with hyperlipidemia 4 A 42-year-old female with Factor V Leiden mutation on warfarin

2 Venous thrombosis is the result of inflammation to a vein, hypercoagulability, venous stasis, or a combination of the three, known as Virchow triad. Bed rest and hip fracture are two major risk factors for the development of a thrombosis. While the other options present risk factors (cigarette smoking, drug abuse, and clotting disorders), the combination of the two (venous stasis and vessel injury) results in greatest risk for thrombus development.

A 22-year-old primigravida is admitted to the hospital in labor. After performing a vaginal examination, the nurse determines that the client's cervix is dilated 2 cm and 80% effaced and that the presenting part is at 0 station. What is the location of the presenting part? 1 Entering the vagina 2 Floating within the bony pelvis 3 At the level of the ischial spines 4 Above the level of the ischial spines

3 The ischial spines are used as landmarks in relation to the fetus's head, because they reflect the progression of labor; 0 station indicates that the presenting part is at the ischial spines. When the head enters the vagina it is below the ischial spines and its position is designated with positive numbers (+1 to +4). When the presenting part is floating, the fetus is at -5 station. A position above the ischial spines is designated by a minus number (-1 to -4).

When palpating a client's fundus on the second postpartum day, the nurse determines that it is above the umbilicus and displaced to the right. What does the nurse conclude? 1 There is a slow rate of involution. 2 There are retained placental fragments. 3 The bladder has become overdistended. 4 The uterine ligaments are overstretched.

3 A distended bladder will displace the fundus upward and laterally to the right. A slow rate of involution is manifested by slow contractions and uterine descent into the pelvis. If retained placental fragments were present, the uterus would be boggy in addition to being displaced, and vaginal bleeding would be heavy. From this assessment the nurse cannot make a judgment regarding overstretched uterine ligaments.

While the nurse is caring for a client in active labor whose fetus is at station 0, the client's membranes rupture spontaneously. The nurse determines that the fluid is clear and odorless. What should the nurse do next? 1 Change the bedding. 2 Notify the practitioner. 3 Assess the fetal heart rate (FHR). 4 Obtain the client's blood pressure.

3 The FHR will reflect how the fetus tolerated the rupture of the membranes; if there is compression of the cord, it will be reflected in a change in the FHR. Although the client's comfort is important, it is not the priority. Although the practitioner should be notified, it is not the priority. Blood pressure is not influenced by rupture of the membranes.

The nurse instructs a pregnant client regarding fetal growth and development. Which statement indicates that the client requires further teaching? 1 "The fetus keeps growing throughout pregnancy." 2 "The fetus may be underweight if it's exposed to smoke." 3 "The fetus gets nutrients from the amniotic fluid." 4 "The fetus gets oxygen from blood in the placenta."

3 The amniotic fluid provides protection, not nutrition; the fetus depends on the placenta, along with the umbilical blood vessels, for nutrients and oxygen. The statements that the fetus keeps growing throughout pregnancy, that it may be underweight if exposed to smoke, and that it gets oxygen from blood in the placenta all indicate that the client understands the teaching.

A client is scheduled for a sonogram at 36 weeks' gestation. Shortly before the test she tells the nurse that she is experiencing severe abdominal pain. Assessment reveals heavy vaginal bleeding, a drop in blood pressure, and an increased pulse rate. Which complication does the nurse suspect? 1 Hydatidiform mole 2 Vena cava syndrome 3 Marginal placenta previa 4 Complete abruptio placentae

4 Severe pain accompanied by bleeding at term or close to it is symptomatic of complete premature detachment of the placenta (abruptio placentae). A hydatidiform mole is diagnosed before 36 weeks' gestation; it is not accompanied by severe pain. There is no bleeding with vena cava syndrome. Bleeding caused by placenta previa should not be painful.

During a health fair, a male client with emphysema tells the nurse that he fatigues easily. Assessment reveals marked clubbing of the fingernails and an increased anteroposterior chest diameter. Which instruction is best to provide the client? A."Pace your activities and schedule rest periods." B."Increase the amount of oxygen you use at night." C."Obtain medical evaluation for antibiotic therapy." D."Reduce your intake of fluids containing caffeine."

A Manifestations of emphysema include an increase in AP diameter (referred to as a barrel chest), nail bed clubbing, and fatigue. The nurse can provide instructions to promote energy management, such as pacing activities and scheduling rest periods (A). (B) may result in a decreased drive to breathe. The client is not exhibiting any symptoms of infection, so (C) is not necessary. (D) is less beneficial than (A).

29. The physician has prescribed salmeterol (Serevent) for a patient with asthma. In reviewing the use of dry powder inhalers (DPIs) with the patient, the nurse should provide which of the following instructions? A. "Close lips tightly around the mouthpiece and breathe in deeply and quickly." B. "To administer a DPI, you must use a spacer that holds the medicine so that you can inhale it." C. "Hold the inhaler several inches in front of your mouth and breathe in slowly, holding the medicine as long as possible." D. "You will know you have correctly used the DPI when you taste or sense the medicine going into your lungs."

A. "Close lips tightly around the mouthpiece and breathe in deeply and quickly." Dry powder inhalers do not require spacer devices. The patient should be instructed to breathe in deeply and quickly to ensure medicine moves down deeply into lungs. The patient may not taste or sense the medicine going into the lungs.

29. Which of the following instructions are most appropriate in the home management of a patient who has undergone surgery for oral cancer? A. "You should drink plenty of fluids and eat foods you enjoy." B. "It is normal to have some leakage of saliva from the suture line." C. "Lying in a prone position helps decrease swelling at the suture line." D. "You should avoid foods high in protein while your suture line is healing."

A. "You should drink plenty of fluids and eat foods you enjoy." For patients who have undergone treatment for head and neck cancers, maintaining adequate nutrition is a challenge. The nurse encourages the patient to increase fluids to prevent dehydration and liquefy secretions. These patients are more likely to eat foods that they enjoy and can tolerate.

32. Which of the following test results identify that a patient with an asthma attack is responding to treatment? A. A decreased exhaled nitric oxide B. An increase in CO2 levels C. A decrease in white blood cell count D. An increase in serum bicarbonate levels

A. A decreased exhaled nitric oxide. Nitric oxide levels are increased in the breath of people with asthma. A decrease in the exhaled nitric oxide concentration suggests that the treatment may be decreasing the lung inflammation associated with asthma.

8. While obtaining the admission assessment data, which of the following characteristics would a nurse expect a patient with anemia to report? A. Palpitations B. Blurred vision C. Increased appetite D. Feeling of warm flushing sensation

A. Palpitations Patients experiencing moderate anemia (hemoglobin [Hb] 6 to 10 g/dL) may experience dyspnea (shortness of breath), palpitations, diaphoresis (profound perspiration) with exertion, and chronic fatigue. Blurred vision is associated in patients experiencing profound anemia states. Anorexia is common in patients with severe anemia, as well. Patients with anemia often appear pale and complain of feeling cold because of compensatory vasoconstriction of the subcutaneous capillaries.

45. The nurse reviews pursed lip breathing with a patient newly diagnosed with emphysema. The nurse reinforces that this technique will assist respiration by which of the following mechanisms? A. Preventing bronchial collapse and air trapping in the lungs during exhalation B. Increasing the respiratory rate and giving the patient control of respiratory patterns C. Loosening secretions so that they may be coughed up more easily D. Promoting maximal inhalation for better oxygenation of the lungs

A. Preventing bronchial collapse and air trapping in the lungs during exhalation The focus of pursed lip breathing is to slow down the exhalation phase of respiration, which decreases bronchial collapse and subsequent air trapping in the lungs during exhalation.

During assessment of a client in the intensive care unit, the nurse notes that the client's ARE CLEAR UPON AUSCULTATION, but jugular vein distention and muffled heart sounds are present. Which intervention should the nurse implement? A.Prepare the client for a pericardial tap. B.Administer intravenous furosemide (Lasix). C.Assist the client to cough and breathe deeply. D.Instruct the client to restrict the oral fluid intake.

A The client is exhibiting symptoms of cardiac tamponade, a collection of fluid in the pericardial sac that results in a reduction in cardiac output, which is a potentially fatal complication of pericarditis. Treatment for tamponade is a pericardial tap (A). Lasix IV is not indicated for treatment of pericarditis (B). Because the client's breath sounds are clear, (C) is not a priority. Fluids are frequently increased (D) in the initial treatment of tamponade to compensate for the decrease in cardiac output, but this is not the same priority as (A).

A client with hypertension has been receiving ramipril (Altace), 5 mg PO, daily for 2 weeks and is scheduled to receive a dose at 0900. At 0830, the client's blood pressure is 120/70 mm Hg. Which action should the nurse take? A.Administer the prescribed dose at the scheduled time. B.Hold the dose and contact the health care provider. C.Hold the dose and recheck the blood pressure in 1 hour. D.Check the health care provider's prescription to clarify dose.

A The client's blood pressure is within normal limits, indicating that the ramipril, an antihypertensive, is having the desired effect and should be administered (A). (B and C) would be appropriate if the client's blood pressure was excessively low (<100 mm Hg systolic) or if the client were exhibiting signs of hypotension such as dizziness. This prescribed dose is within the normal dosage range, as defined by the manufacturer; therefore, (D) is not necessary.

The nurse is preparing a teaching plan for a group of healthy adults. Which individual is most likely to maintain optimum health? A.A teacher whose blood glucose levels average 126 mg/dL daily with oral antidiabetic drugs B.An accountant whose blood pressure averages 140/96 mm Hg and who says he does not have time to exercise C.A stock broker whose total serum cholesterol level dropped to 290 mg/dL with diet modifications D.A recovering IV heroin user who contracted hepatitis more than 10 years ago

A The diabetic teacher (A) has assumed responsibility for self-care, so among those listed, is the most likely to maintain optimum health. (B) has expressed a lack of interest in health promotion. (C) continues to demonstrate a high-risk cholesterol level despite a reported attempt at dietary modifications. Previous IV drug use and a history of hepatitis (D) make this individual a health risk despite the fact that the individual is in recovery.

A client diagnosed with angina pectoris complains of chest pain while ambulating in the hallway. Which action should the nurse implement first? A.Support the client to a sitting position. B.Ask the client to walk slowly back to the room. C.Administer a sublingual nitroglycerin tablet. D.Provide oxygen via nasal cannula.

A The nurse should safely assist the client to a resting position (A) and then perform (C and D). The client must cease all activity immediately, which will decrease the oxygen requirement of the myocardial muscle. After these interventions are implemented, the client can be escorted back to the room via wheelchair or stretcher (B).

Which content about self-care should the nurse include in the teaching plan of a female client who has genital herpes? (Select all that apply.) A.Encourage annual physical and Pap smear. B.Take antiviral medication as prescribed. C.Use condoms to avoid transmission to others. D.Warm sitz baths may relieve itching. E.Use Nystatin suppositories to control itching. F.Use a douche with weak vinegar solution to decrease itching.

A, B, C, D The nurse should include (A, B, C, and D) in the teaching plan of a female client with genital herpes. (E) is specific for Candida infections, and (F) is used to treat Trichomonas.

The nurse is conducting an osteoporosis screening clinic at a health fair. What information should the nurse provide to individuals who are at risk for osteoporosis? (Select all that apply.) A.Encourage alcohol and smoking cessation. B.Suggest supplementing diet with vitamin E. C.Promote regular weight-bearing exercises. D.Implement a home safety plan to prevent falls. E.Propose a regular sleep pattern of 8 hours nightly.

A, C, D (A, C, and D) are factors that decrease the risk for developing osteoporosis. Vitamin D and calcium are important supplements to aid in the decrease of bone loss (B). Regular sleep patterns are important to overall health but are not identified with a decreasing risk for osteoporosis (E).

57. When admitting a patient with the diagnosis of asthma exacerbation, the nurse will assess for which of the following potential triggers? (Select all that apply.) A. Exercise B. Allergies C. Emotional stress D. Decreased humidity

A,B,C Although the exact mechanism of asthma is unknown, there are several triggers that may precipitate an attack. These include allergens, exercise, air pollutants, respiratory infections, drug and food additives, psychologic factors, and GERD.

55. During admission of a patient diagnosed with non-small cell carcinoma of the lung, the nurse questions the patient related to a history of which of the following risk factors for this type of cancer? (Select all that apply.) A. Asbestos exposure B. Cigarette smoking C. Exposure to uranium D. Chronic interstitial fibrosis

A,B,C Non-small carcinoma is associated with cigarette smoking and exposure to environmental carcinogens, including asbestos and uranium. Chronic interstitial fibrosis is associated with the development of adenocarcinoma of the lung.

52. When assessing a patient's sleep-rest pattern related to respiratory health, the nurse would ask if the patient: (Select all that apply.) A. Has trouble falling asleep B. Awakens abruptly during the night C. Sleeps more than 8 hours per night D. Has to sleep with the head elevated

A,B,D The patient with sleep apnea may have insomnia and/or abrupt awakenings. Patients with cardiovascular disease (e.g., heart failure that may affect respiratory health) may need to sleep with the head elevated on several pillows (orthopnea). Sleeping more than 8 hours per night is not indicative of impaired respiratory health.

56. When admitting a 45-year-old female with a diagnosis of pulmonary embolism, the nurse will assess the patient for which of the following risk factors? (Select all that apply.) A. Obesity B. Pneumonia C. Hypertension D. Cigarette smoking

A,C,D Research has demonstrated an increased risk of pulmonary embolism in women associated with obesity, heavy cigarette smoking, and hypertension. Other risk factors include immobilization, surgery within the last 3 months, stroke, history of DVT, and malignancy.

54. To promote airway clearance in a patient with pneumonia, the nurse instructs the patient to do which of the following? (Select all that apply.) A. Splint the chest when coughing B. Maintain a semi-Fowler's position C. Maintain adequate fluid intake D. Instruct patient to cough at end of exhalation

A,C,D The nurse should instruct the patient to splint the chest while coughing. This will reduce discomfort and allow for a more effective cough. Maintaining adequate fluid intake liquefies secretions, allowing easier expectoration. Coughing at the end of exhalation promotes a more effective cough. The patient should be positioned in an upright sitting position (high-Fowler's) with head slightly flexed.

19. Which of the following statements made by a nurse would indicate proper teaching principles regarding feeding and tracheostomies? A. "Follow each spoon of food consumed with a drink of fluid." B. "Thin your foods to a liquid consistency whenever possible." C. "Tilt your chin forward toward the chest when swallowing your food." D. "Make sure your cuff is overinflated before eating if you have swallowing problems."

C. "Tilt your chin forward toward the chest when swallowing your food." A nurse should instruct a patient to tilt the chin toward the chest, which will close the glottis and allow food to enter the normal passageway. Ideally, foods should be of a thick consistency to enable effective swallowing and reduce the risk of aspiration. Overinflation of the cuff causes swallowing difficulties. Fluids should be consumed in small amounts after swallowing to prevent the risk of aspiration.

6. When preparing to administer an ordered blood transfusion, the nurse selects which of the following intravenous solutions to use when priming the blood tubing? A. 5% dextrose in water B. Lactated Ringer's C. 0.9% sodium chloride D. 0.45% sodium chloride

C. 0.9% sodium chloride The blood set should be primed before the transfusion with 0.9% sodium chloride, also known as normal saline. It is also used to flush the blood tubing after the infusion is complete to ensure the patient receives blood that is left in the tubing when the bag is empty.

43. The nurse evaluates that nursing interventions to promote airway clearance in a patient admitted with COPD are successful based on which of the following findings? A. Absence of dyspnea B. Improved mental status C. Effective and productive coughing D. PaO2 within normal range for the patient

C. Effective and productive coughing The issue of the question is airway clearance, which is most directly evaluated as successful if the patient can engage in effective and productive coughing.

46. Nursing assessment findings of jugular vein distention and pedal edema would be indicative of which of the following complications of emphysema? A. Acute respiratory failure B. Pulmonary edema caused by left-sided heart failure C. Fluid volume excess secondary to cor pulmonale D. Secondary respiratory infection

C. Fluid volume excess secondary to cor pulmonale Cor pulmonale is a right-sided heart failure caused by resistance to right ventricular outflow due to lung disease. With failure of the right ventricle, the blood emptying into the right atrium and ventricle would be slowed, leading to jugular venous distention and pedal edema.

23. Which of the following positions is most appropriate for the nurse to place a patient experiencing an asthma exacerbation? A. Supine B. Lithotomy C. High-Fowler's D. Reverse Trendelenburg

C. High-Fowler'sThe patient experiencing an asthma attack should be placed in high-Fowler's position to allow for optimal chest expansion and enlist the aid of gravity during inspiration.

8. Which of the following clinical manifestations would the nurse expect to find during assessment of a patient admitted with pneumococcal pneumonia? A. Hyperresonance on percussion B. Fine crackles in all lobes on auscultation C. Increased vocal fremitus on palpation D. Vesicular breath sounds in all lobes

C. Increased vocal fremitus on palpation. A typical physical examination finding for a patient with pneumonia is increased vocal fremitus on palpation. Other signs of pulmonary consolidation include dullness to percussion, bronchial breath sounds, and crackles in the affected area.

A tornado warning alarm has been activated at the local hospital. Which action should the charge nurse working on a surgical unit implement first? A.Instruct the nursing staff to close all window blinds and curtains in clients' rooms. B.Move clients and visitors into the hallways and close all doors to clients' rooms. C.Visually confirm the location of the tornado by checking the windows on the unit. D.Assist all visitors with evacuation down the stairs in a calm and orderly manner.

B In the event of a tornado, all persons should be moved into the hallways, away from windows, to prevent flying debris from causing injury (B). Although (A) may help decrease the amount of flying debris, it is not safe to leave clients in rooms with closed blinds; (B) is a higher priority at this time. Hospital staff should stay away from windows to avoid injury and should focus on client evacuation into hallways rather than (C). (D) is not the first action that should be taken.

Which condition should the nurse anticipate as a potential problem in a female client with a neurogenic bladder? A.Stress incontinence B.Infection C.Painless gross hematuria D.Peritonitis

B Infection (B) is the major complication resulting from stasis of urine and subsequent catheterization. (A) is the involuntary loss of urine through an intact urethra as a result of a sudden increase in intraabdominal pressure. (C) is the most common symptom of bladder cancer. (D) is the most common and serious complication of peritoneal dialysis.

The nurse is interviewing a client who is taking interferon-alfa-2a (Roferon-A) and ribavirin (Virazole) combination therapy for hepatitis C. The client reports experiencing overwhelming feelings of depression. Which action should the nurse implement first? A.Recommend mental health counseling. B.Review the medication actions and interactions. C.Assess for the client's daily activity level. D.Provide information regarding a support group.

B Interferon-alfa-2a and ribavirin combination therapy can cause severe depression (B); therefore, it is most important for the nurse to review the medication effects and report these to the health care provider. (A, C, and D) might be implemented after the physiologic aspects of the situation have been assessed.

2. The nurse is caring for a patient with COPD and pneumonia who has an order for arterial blood gases to be drawn. Which of the following is the minimum length of time the nurse should plan to hold pressure on the puncture site? A. 2 minutes B. 5 minutes C. 10 minutes D. 15 minutes

B. 5 minutes Following obtaining an arterial blood gas, the nurse should hold pressure on the puncture site for 5 minutes by the clock to be sure that bleeding has stopped. An artery is an elastic vessel under higher pressure than veins, and significant blood loss or hematoma formation could occur if the time is insufficient.

4. The nurse is caring for a patient who is to receive a transfusion of two units of packed red blood cells. After obtaining the first unit from the blood bank, the nurse would ask which of the following health team members in the nurses' station to assist in checking the unit before administration? A. Unit secretary B. Another registered nurse C. A physician's assistant D. A phlebotomist

B. Another registered nurseBefore hanging a transfusion, the registered nurse must check the unit with another RN or with a licensed practical (vocational) nurse, depending on agency policy.

13. If a patient with an uncuffed tracheostomy tube coughs violently during suctioning and dislodges the tracheostomy tube, a nurse should first A. call the physician. B. attempt to reinsert the tracheostomy tube. C. position the patient in a lateral position with the neck extended. D. cover the stoma with a sterile dressing and ventilate the patient with a manual bag-mask until the physician arrives.

B. attempt to reinsert the tracheostomy tube.Retention sutures may be grasped (if present) and the tracheostomy opening spread, or a hemostat may be used to spread the opening. The obturator is inserted into the replacement tube (one size smaller than the original tube), lubricated with saline solution, and inserted into the stoma at a 45-degree angle to the neck. If the attempt is successful, the obturator tube should immediately be removed.

1. A nurse is reviewing the hematologic test results for a patient in whom the hematocrit (Hct) is reported at a reading of 30%. Based on this result, the nurse should interpret that the patient A. is susceptible to bleeding disorders. B. has fewer red blood cells than normal. C. is experiencing an inflammatory response. D. is experiencing an acute hemolytic crisis.

B. has fewer red blood cells than normal.The Hct is the measure of the volume of red blood cells in whole blood expressed as a percentage. This test is useful in the diagnosis of anemia, polycythemia, and abnormal hydration states. Patients who are susceptible to bleeding disorders likely will have a low platelet count. The inflammatory response may best be evaluated by examination of results that include the white blood cell count with differential analysis. Acute hemolytic crisis develops in patients receiving blood components in which incompatibility occurs or in patients with bleeding disorders or conditions that promote cellular damage, such as damage associated with shock.

18. Nursing interventions for the patient with aplastic anemia are directed toward the prevention of the complications of A. fatigue and dyspnea. B. hemorrhage and infection. C. thromboemboli and gangrene. D. cardiac arrhythmias and heart failure.

B. hemorrhage and infection. Hemorrhage from thrombocytopenia and infection from neutropenia are the greatest risks for the patient with aplastic anemia. The patient will experience fatigue from anemia, but bleeding and infection are the major causes of death in aplastic anemia.

3. In preparing the preoperative teaching plan for a patient who is to undergo a total laryngectomy, a nurse should give highest priority to the A. tracheostomy being in place for 2 to 3 days. B. patient's not being able to speak normally again. C. insertion of a gastrostomy feeding tube during surgery. D. patient's not being able to perform deep-breathing exercises.

B. patient's not being able to speak normally again. Patients who have a total laryngectomy have a permanent tracheostomy and will need to learn how to speak using alternative methods, such as an artificial larynx. The tracheostomy will be permanent to allow normal breathing patterns and air exchange. After surgery, the patient's nutrition is supplemented with enteral feedings, and when the patient can swallow secretions, oral feedings can begin. Deep-breathing exercises should be performed with the patient at least every 2 hours to prevent further pulmonary complications.

19. A patient with hemophilia is hospitalized with acute knee pain and swelling. An appropriate nursing intervention for the patient includes A. wrapping the knee with an elastic bandage. B. placing the patient on bed rest and applying ice to the joint. C. gently performing range-of-motion (ROM) exercises to the knee to prevent adhesions. D. administering nonsteroidal anti-inflammatory drugs (NSAIDs) as needed for pain.

B. placing the patient on bed rest and applying ice to the joint. During an acute bleeding episode in a joint, it is important to totally rest the involved joint and slow bleeding with application of ice. Drugs that decrease platelet aggregation, such as aspirin or NSAIDs, should not be used for pain. As soon as bleeding stops, mobilization of the affected area is encouraged with range-of-motion (ROM) exercises and physical therapy.

36. The resurgence in TB resulting from the emergence of multidrug-resistant strains of Mycobacterium tuberculosis is primarily the result of A. a lack of effective means to diagnose TB. B. poor compliance with drug therapy in patients with TB. C. the increased population of immunosuppressed individuals with AIDS. D. indiscriminate use of antitubercular drugs in treatment of other infections.

B. poor compliance with drug therapy in patients with TB. Drug-resistant strains of TB have developed because TB patients' compliance to drug therapy has been poor and there has been general decreased vigilance in monitoring and follow-up of TB treatment. Antitubercular drugs are almost exclusively used for TB infections. TB can be effectively diagnosed with sputum cultures. The incidence of TB is at epidemic proportions in patients with HIV, but this does not account for drug-resistant strains of TB.

2. A patient admitted to the emergency department with tension pneumothorax and mediastinal shift following an automobile crash is most likely to exhibit A. bradycardia. B. severe hypotension. C. mediastinal flutter. D. a sucking chest wound.

B. severe hypotension. Mediastinal shift may cause compression of the lung in the direction of the shift and compression, traction, torsion, or kinking of the great vessels. Blood return to the heart is dangerously impaired and causes a subsequent decrease in cardiac output and blood pressure. Tachycardia is a clinical manifestation of tension pneumothorax. An uncovered opened pneumothorax is associated with a sucking chest wound and mediastinal flutter.

25. A 75-year-old obese patient who is snoring loudly and having periods of apnea several times each night is most likely experiencing A. narcolepsy. B. sleep apnea. C. sleep deprivation. D. paroxysmal nocturnal dyspnea.

B. sleep apnea. Sleep apnea is most common in obese patients. Typical symptoms include snoring and periods of apnea. Narcolepsy is when a patient falls asleep unexpectedly. Sleep deprivation could result from sleep apnea. Paroxysmal nocturnal dyspnea occurs when a patient has shortness of breath during the night.

40. Select all that apply. Atelectasis can be caused by A. long-term smoking. B. inadequate surfactant. C. localized airway obstruction. D. an increase in lung expansion. E. an increase in elastic recoil.

BCE The collapse of lung tissue has several causes, including reduced lung expansion, localized airway obstruction, inadequate surfactant, and an increase in elastic recoil. Smoking, although harmful, does not in itself cause atelectasis.

21. Select all that apply. Which of the following are significant risk factors for leukemia? A. Being a longtime smoker B. Employment in an oil refinery C. History of hemophilia in parent D. Having Down syndrome E. Having a twin brother with leukemia F. Treatment with an alkylating agent = 3 years ago

BDEF Exposure to chemical agents, treatment with alkylating cancer drugs, leukemia in a sibling, and the patient's having Down syndrome are all risk factors for leukemia.

In assessing a client with an arteriovenous (AV) shunt who is scheduled for dialysis today, the nurse notes the ABSENCE of a thrill or bruit at the shunt site. What action should the nurse take? A.Advise the client that the shunt is intact and ready for dialysis as scheduled. B.Encourage the client to keep the shunt site elevated above the level of the heart. C.Notify the health care provider of the findings immediately. D.Flush the site at least once with a heparinized saline solution.

C Absence of a thrill or bruit indicates that the shunt may be obstructed. The nurse should notify the health care provider (C) so that intervention can be initiated to restore function of the shunt. (A) is incorrect. (B) will not resolve the obstruction. An AV shunt is internal and cannot be flushed (D) without access using special needles.

A postoperative client receives a Schedule II opioid analgesic for pain. Which assessment finding requires the most immediate intervention by the nurse? A.Hypoactive bowel sounds with abdominal distention B.Client reports continued pain of 8 on a 10-point scale C.Respiratory rate of 12 breaths/min, with O2 saturation of 85% D.Client reports nausea after receiving the medication

C Administration of a Schedule II opioid analgesic can result in respiratory depression (C), which requires immediate intervention by the nurse to prevent respiratory arrest. (A, B, and D) require action by the nurse but are of less priority than (C).

During the change of shift report, the charge nurse reviews the infusions being received by clients on the oncology unit. The client receiving which infusion should be assessed first? A.Continuous IV infusion of magnesium B.One-time infusion of albumin C.Continuous epidural infusion of morphine D.Intermittent infusion of IV vancomycin

C All four of these clients have the potential to have significant complications. The client with the morphine epidural infusion (C) is at highest risk for respiratory depression and should be assessed first. (A) can cause hypotension. The client receiving (B) is at lowest risk for serious complications. Although (D) can cause nephrotoxicity and phlebitis, these problems are not as immediately life threatening as (C).

The nurse is completing an admission interview for a client with Parkinson's disease. Which question will provide additional information about manifestations that the client is likely to experience? A."Have you ever experienced any paralysis of your arms or legs?" B."Do you have frequent blackout spells?" C."Have you ever been frozen in one spot, unable to move?" D."Do you have headaches, especially ones with throbbing pain?"

C Clients with Parkinson's disease frequently experience difficulty in initiating, maintaining, and performing motor activities. They may even experience being rooted to the spot and unable to move (C). Parkinson's disease does not typically cause (A, B, or D).

During report, the nurse learns that a client with tumor lysis syndrome is receiving an IV infusion containing insulin. Which assessment should the nurse complete first? A.Review the client's history for diabetes mellitus. B.Observe the extremity distal to the IV site. C.Monitor the client's serum potassium and blood glucose levels. D.Evaluate the client's oxygen saturation and breath sounds.

C Clients with tumor lysis syndrome may experience hyperkalemia, requiring the addition of insulin to the IV solution to reduce the serum potassium level. It is most important for the nurse to monitor the client's serum potassium and blood glucose levels to ensure that they are not at dangerous levels (C). (A, B, and D) provide valuable assessment data but are of less priority than (C).

When educating a client after a total laryngectomy, which instruction would be most important for the nurse to include in the discharge teaching? A.Recommend that the client carry suction equipment at all times. B.Instruct the client to have writing materials with him at all times. C.Tell the client to carry a medical alert card that explains his condition. D.Caution the client not to travel outside the United States alone.

C Neck breathers carry a medical alert card (C) that notifies health care personnel of the need to use mouth to stoma breathing in the event of a cardiac arrest in this client. Mouth to mouth resuscitation will not establish a patent airway. (A and D) are not necessary. There are many alternative means of communication for clients who have had a laryngectomy; dependence on writing messages (B) is probably the least effective.

A client in the emergency department is bleeding profusely from a gunshot wound to the abdomen. In what position should the nurse immediately place the client to promote maintenance of the client's blood pressure above a systolic pressure of 90 mm Hg? A.Place the client in a 45-degree Trendelenburg position to promote cerebral blood flow. B.Turn the client prone to place pressure on the abdominal wound to help staunch the bleeding. C.Maintain the client in a supine position to reduce diaphragmatic pressure and visualize the wound. D.Put the client on the right side to apply pressure to the liver and spleen to stop hemorrhaging.

C Placing the client in a supine position (C) reduces diaphragmatic pressure, thereby enhancing oxygenation, and allows for visualization of the abdominal wound. (A) compromises diaphragmatic expansion and inhibits pressoreceptor activity. (B) places the client at risk of evisceration of the abdominal wound and increased bleeding. (D) will not stop internal bleeding in the liver and spleen caused by the gunshot wound.

When a nurse assesses a client receiving total parenteral nutrition (TPN), which laboratory value is most important for the nurse to monitor regularly? A.Albumin B.Calcium C.Glucose D.Alkaline phosphatase

C TPN solutions contain high concentrations of glucose, so the blood glucose level is often monitored as often as q6h because of the risk for hyperglycemia (C). (A) is monitored periodically because an increase in the albumin level, a serum protein, is generally a desired effect of TPN. (B) may be added to TPN solutions, but calcium imbalances are not generally a risk during TPN administration. (D) may be decreased in the client with malnutrition who receives TPN, but abnormal values, reflecting liver or bone disorders, are not a common complication of TPN administration.

The nurse is caring for a critically ill client with cirrhosis of the liver who has a nasogastric tube draining bright red blood. The nurse notes that the client's serum hemoglobin and hematocrit levels are decreased. Which additional change in laboratory data should the nurse expect? A.Increased serum albumin level B.Decreased serum creatinine C.Decreased serum ammonia level D.Increased liver function test results

C The breakdown of glutamine in the intestine and the increased activity of colonic bacteria from the digestion of proteins increase ammonia levels in clients with advanced liver disease, so removal of blood, a protein source, from the intestine results in a reduced level of ammonia (C). (A, B, and D) will not be significantly affected by the removal of blood.

39. Select all that apply. Which of the following are clinical manifestations of tension pneumothorax? A. Midline trachea B. Severe hypertension C. Progressive cyanosis D. A loud bruit on affected side E. Asymmetrical chest wall movement F. Subcutaneous emphysema in the neck

C,E, F The indicators of tension pneumothorax are asymmetrical chest wall movement, severe hypotension, subcutaneous emphysema in the neck and upper chest, and progressive cyanosis.

51. Which of the following statements made by a patient with COPD indicates a need for further education regarding the use of an ipratropium inhaler? A. "I should rinse my mouth following the two puffs to get rid of the bad taste." B. "I should wait at least 1 to 2 minutes between each puff of the inhaler." C. "If my breathing gets worse, I should keep taking extra puffs of the inhaler until I can breathe more easily." D. "Because this medication is not fast-acting, I cannot use it in an emergency if my breathing gets worse.

C. "If my breathing gets worse, I should keep taking extra puffs of the inhaler until I can breathe more easily." The patient should not take extra puffs of the inhaler at will to make breathing easier. Excessive treatment could trigger paradoxical bronchospasm, which would worsen the patient's respiratory status.

13. If a patient with arthritis develops iron-deficiency anemia, a nurse should ask about the patient's use of A. alcoholic beverages. B. stool softeners and laxatives. C. caffeinated foods and beverages. D. NSAIDs.

D. NSAIDs. NSAIDs decrease the level of vitamin C, which aids in the absorption of iron. These drugs also compete with folate and vitamin K and may cause gastritis. Excessive alcoholic beverage consumption can cause stomach irritation; alcohol would not be directly related to iron-deficiency anemia unless bleeding ulcers or gastritis were to occur. NSAID consumption, not stool softeners and laxative use, would be suspect for iron-deficiency anemia. Caffeinated foods and beverages can cause gastric irritation and discomfort but are not associated with iron-deficiency anemia.

A central venous catheter has been inserted via a jugular vein, and a radiograph has confirmed placement of the catheter. A prescription has been received for a medication STAT, but IV fluids have not yet been started. Which action should the nurse take prior to administering the prescribed medication? A.Assess for signs of jugular venous distention. B.Obtain the needed intravenous solution. C.Flush the line with heparinized solution. D.Flush the line with normal saline.

D Medication can be administered via a central line without additional IV fluids. The line should first be flushed with a normal saline solution (D) to ensure patency. Insufficient evidence exists on the effectiveness of flushing catheters with heparin (C). (A) will not affect the decision to administer the medication and is not a priority. Administration of the medication STAT is of greater priority than (B).

The nurse is assessing a client who presents with jaundice. Which assessment finding is most important for the nurse to follow up? A.Urine specific gravity of 1.03 B.Frothy, tea-colored urine C.Clay-colored stools D.Elevated serum amylase and lipase levels

D Obstructive cholelithiasis and alcoholism are the two major causes of pancreatitis, and elevated serum amylase and lipase levels (D) indicate pancreatic injury. (A) is a normal finding. (B and C) are expected findings related to jaundice.

The nurse receives the client's next scheduled bag of TPN labeled with the additive NPH insulin. Which action should the nurse implement? A.Hang the solution at the current rate. B.Refrigerate the solution until needed. C.Prepare the solution with new tubing. D.Return the solution to the pharmacy.

D Only regular insulin is administered by the IV route, so the TPN solution containing NPH insulin should be returned to the pharmacy (D). (A, B, and C) are not indicated because the solution should not be administered.

A client with type 2 diabetes takes metformin (Glucophage) daily. The client is scheduled for major surgery requiring general anesthesia the next day. The nurse anticipates which approach to manage the client's diabetes best while the client is NPO during the perioperative period? A.NPO except for metformin and regular snacks B.NPO except for oral antidiabetic agent C.Novolin N insulin subcutaneously twice daily D.Regular insulin subcutaneously per sliding scale

D Regular insulin dosing based on the client's blood glucose levels (sliding scale) is the best method to achieve control of the client's blood glucose while the client is NPO and coping with the major stress of surgery (D). (A) increases the risk of vomiting and aspiration. (B and C) provide less precise control of the blood glucose level.

A client with cirrhosis develops increasing pedal edema and ascites. Which dietary modification is most important for the nurse to teach this client? A.Avoid high-carbohydrate foods. B.Decrease intake of fat-soluble vitamins. C.Decrease caloric intake. D.Restrict salt and fluid intake.

D Salt and fluid restrictions are the first dietary modifications for a client who is retaining fluid as manifested by edema and ascites (D). (A, B, and C) will not affect fluid retention.

A client with alcohol-related liver disease is admitted to the unit. Which prescription should the nurse call the health care provider about for reverification for this client? A.Vitamin K1 (AquaMEPHYTON), 5 mg IM daily B.High-calorie, low-sodium diet C.Fluid restriction to 1500 mL/day D.Pentobarbital (Nembutal sodium) at bedtime for rest

D Sedatives such as Nembutal (D) are contraindicated for clients with liver damage and can have dangerous consequences. (A) is often prescribed because the normal clotting mechanism is damaged. (B) is needed to help restore energy to the debilitated client. Sodium is often restricted because of edema. Fluids are restricted (C) to decrease ascites, which often accompanies cirrhosis, particularly in the later stages of the disease.

The nurse is assessing a 75-year-old client for symptoms of hyperglycemia. Which symptom of hyperglycemia is an OLDER adult most likely to exhibit? A.Polyuria B.Polydipsia C.Weight loss D.Infection

D Signs and symptoms of hyperglycemia in older adults may include fatigue, infection (D), and evidence of neuropathy (e.g., sensory changes). The nurse needs to remember that classic signs and symptoms of hyperglycemia, such as (A, B, and C) and polyphagia, may be absent in older adults.

4. The nurse is teaching a patient who is to undergo bone marrow aspiration. Which of the following statements made by the nurse would indicate correct instruction regarding the site for the aspiration procedure? A. "The health care provider will perform the aspiration by needle to the femur." B. "The health care provider will perform the aspiration by needle to the scapula." C. "The health care provider will perform the aspiration by needle to the antecubital fossa." D. "The health care provider will perform the aspiration by needle to the posterior iliac crest."

D. "The health care provider will perform the aspiration by needle to the posterior iliac crest." Bone marrow samples are commonly taken from the posterior iliac crest or, as an alternative, the sternum may be aspirated. These sites provide relative ease in accessing the bone marrow via the biopsy needle. The antecubital fossa, femur, and scapula do not allow access to bone marrow while also providing reduced risk of harm to the patient.

5. Before starting a transfusion of packed red blood cells for an anemic patient, the nurse would arrange for a peer to monitor his or her other assigned patients for how many minutes when the nurse begins the transfusion? A. 60 B. 5 C. 30 D. 15

D. 15 As part of standard procedure, the nurse remains with the patient for the first 15 minutes after hanging a blood transfusion. Patients who are likely to have a transfusion reaction will more often exhibit signs within the first 15 minutes that the blood is infusing.

42. The patient has an order for albuterol 5 mg via nebulizer. Available is a solution containing 1 mg/ml. How many milliliters should the nurse use to prepare the patient's dose? A. 0.2 B. 2.5 C. 3.75 D. 5.0

D. 5.0

1. The arterial blood gas (ABG) readings that indicate compensated respiratory acidosis are a PaCO2 of A. 30 mm Hg and bicarbonate level of 24 mEq/L. B. 30 mm Hg and bicarbonate level of 30 mEq/L. C. 50 mm Hg and bicarbonate level of 20 mEq/L. D. 50 mm Hg and bicarbonate level of 30 mEq/L.

D. 50 mm Hg and bicarbonate level of 30 mEq/L. If compensation is present, carbon dioxide and bicarbonate are abnormal (or nearly so) in opposite directions (e.g., one is acidotic and the other alkalotic).

31. The nurse determines that the patient understood medication instructions about the use of a spacer device when taking inhaled medications after hearing the patient state which of the following as the primary benefit? A. "Now I will not need to breathe in as deeply when taking the inhaler medications." B. "This device will make it so much easier and faster to take my inhaled medications." C. "I will pay less for medication because it will last longer." D. "More of the medication will get down into my lungs to help my breathing."

D. "More of the medication will get down into my lungs to help my breathing." A spacer assists more medication to reach the lungs, with less being deposited in the mouth and the back of the throat.

27. A patient with COPD is receiving oxygen at 2 L/min. While in the supine position for a bath, the patient complains of shortness of breath. What is the most appropriate first nursing action? A. Increase the flow of oxygen. B. Perform tracheal suctioning. C. Report this to the physician. D. Assist the patient to Fowler's position.

D. Assist the patient to Fowler's position. Breathing is easier in Fowler's position because it permits greater expansion of the chest cavity. If repositioning does not improve the situation, then oxygenation and physician reporting might be appropriate. The patient would not benefit from tracheal suctioning.

7. Which of the following physical assessment findings in a patient with pneumonia best supports the nursing diagnosis of ineffective airway clearance? A. Oxygen saturation of 85% B. Respiratory rate of 28 C. Presence of greenish sputum D. Basilar crackles

D. Basilar crackles The presence of adventitious breath sounds indicates that there is accumulation of secretions in the lower airways. This would be consistent with a nursing diagnosis of ineffective airway clearance because the patient is retaining secretions.

22. Respiratory acidosis is at highest risk in a patient with A. hypokalemia. B. pulmonary fibrosis. C. salicylate overdose. D. COPD.

D. COPD. Chronic respiratory acidosis is most commonly caused by COPD. Pulmonary fibrosis, hypokalemia, and salicylate overdose do not predispose a patient to respiratory acidosis. Hypokalemia can lead to cardiac dysrhythmias. Salicylate overdose results in central nervous system changes, and pulmonary fibrosis can result in respiratory arrest.

22. The nurse is assigned to care for a patient who has anxiety and an exacerbation of asthma. Which of the following is the primary reason for the nurse to carefully inspect the chest wall of this patient? A. Observe for signs of diaphoresis B. Allow time to calm the patient C. Monitor the patient for bilateral chest expansion D. Evaluate the use of intercostal muscles

D. Evaluate the use of intercostal muscles The nurse physically inspects the chest wall to evaluate the use of intercostal (accessory) muscles, which gives an indication of the degree of respiratory distress experienced by the patient.

14. Which of the following foods is high in iron? A. Citrus fruits B. Milk products C. Yellow vegetables D. Green leafy vegetables

D. Green leafy vegetables Green leafy vegetables are high in iron. Foods cooked in iron pots and foods such as liver (the richest source), oysters, lean meats, kidney beans, whole wheat bread, kale, spinach, egg yolk, turnip tops, beet greens, carrots, apricots, and raisins are also high in iron.

41. When planning patient teaching about emphysema, the nurse understands that the symptoms of emphysema are caused by which of the following? A. Hypertrophy and hyperplasia of goblet cells in the bronchi B. Collapse and hypoventilation of the terminal respiratory unit C. An overproduction of the antiprotease alpha1-antitrypsin D. Hyperinflation of alveoli and destruction of alveolar walls

D. Hyperinflation of alveoli and destruction of alveolar walls In emphysema, there are structural changes that include hyperinflation of alveoli, destruction of alveolar walls, destruction of alveolar capillary walls, narrowing of small airways, and loss of lung elasticity.

5. A nurse is performing assessment for a patient diagnosed with chronic obstructive pulmonary disease (COPD). Which of the following findings should the nurse expect to observe? A. Nonproductive cough B. Prolonged inspiration C. Vesicular breath sounds D. Increased anterior-posterior chest diameter

D. Increased anterior-posterior chest diameter An increased anterior-posterior diameter is a compensatory mechanism experienced by patients with COPD and is caused by air-trapping. Patients with COPD have a productive cough, often expectorating copious amounts of sputum. Because of air-trapping, patients with COPD experience a prolonged expiration because the rate of gas on exhalation takes longer to escape. Chest auscultation for patients with COPD often reveals wheezing, crackles, and other adventitious breath sounds.

2. When assessing lab values on a patient admitted with septicemia, the nurse would expect to find: A. Increased platelets B. Decreased red blood cells C. Decreased erythrocyte sedimentation rate (ESR) D. Increased bands in the WBC differential (shift to the left)

D. Increased bands in the WBC differential (shift to the left) When infections are severe, such as in septicemia, more granulocytes are released from the bone marrow as a compensatory mechanism. To meet the increased demand, many young, immature polymorphonuclear neutrophils (bands) are released into circulation. WBCs are usually reported in order of maturity, with the less mature forms on the left side of a written report. Hence, the term "shift to the left" is used to denote an increase in the number of bands.

10. The blood bank notifies the nurse that the two units of blood ordered for an anemic patient are ready for pick up. The nurse should take which of the following actions to prevent an adverse effect during this procedure? A. Immediately pick up both units of blood from the blood bank. B. Regulate the flow rate so that each unit takes at least 4 hours to transfuse. C. Set up the Y-tubing of the blood set with dextrose in water as the flush solution. D. Infuse the blood slowly for the first 15 minutes of the transfusion.

D. Infuse the blood slowly for the first 15 minutes of the transfusion. Because a transfusion reaction is more likely to occur at the beginning of a transfusion, the nurse should initially infuse the blood at a rate no faster than 2 ml/min and remain with the patient for the first 15 minutes after hanging a unit of blood.

30. Which of the following conditions or factors in a 64-year-old patient diagnosed with head and neck cancer most likely contributed to this health problem? A. Patient's hobby is oil painting. B. Patient's father also had head and neck cancer. C. Patient uses chewing tobacco and drinks beer daily. D. Patient quit school at age 16 and has worked in a butcher shop for more than 40 years.

C. Patient uses chewing tobacco and drinks beer daily. Many environmental risk factors contribute to the development of head and neck cancer, although the actual cause is unknown. There does not appear to be a genetic predisposition to this type of cancer. The two most important risk factors are tobacco and alcohol use, especially in combination. Other risk factors include chewing tobacco, pipe smoking, marijuana use, voice abuse, chronic laryngitis, exposure to industrial chemicals or hardwood dust, and poor oral hygiene.

37. The chronic inflammation of the bronchi characteristic of chronic obstructive pulmonary disease (COPD) results in A. collapse of small bronchioles on expiration. B. permanent, abnormal dilation of the bronchi. C. hyperplasia of mucus-secreting cells and bronchial edema. D. destruction of the elastic and muscular structures of the bronchial wall.

C. hyperplasia of mucus-secreting cells and bronchial edema. Chronic bronchitis is characterized by chronic inflammation of the bronchial lining, with edema and increased mucus production. Collapse of small bronchioles on expiration is common in emphysema, and abnormal dilation of the bronchi because of destruction of the elastic and muscular structures is characteristic of bronchiectasis.

38. The nurse is evaluating whether a patient understands how to safely determine whether a metered dose inhaler is empty. The nurse interprets that the patient understands this important information to prevent medication underdosing when the patient describes which method to check the inhaler? A. Place it in water to see if it floats. B. Shake the canister while holding it next to the ear. C. Check the indicator line on the side of the canister. D. Keep track of the number of inhalations used.

D. Keep track of the number of inhalations used. It is no longer appropriate to see if a canister floats in water or not as research has demonstrated this is not accurate. The best method to determine when to replace an inhaler is by knowing the maximum puffs available per MDI and then replacing when those inhalations have been used.

28. While teaching a patient with asthma about the appropriate use of a peak flow meter, the nurse instructs the patient to do which of the following? A. Use the flow meter each morning after taking medications to evaluate their effectiveness. B. Empty the lungs and then inhale quickly through the mouthpiece to measure how fast air can be inhaled. C. Keep a record of the peak flow meter numbers if symptoms of asthma are getting worse. D. Increase the doses of the long-term control medication if the peak flow numbers decrease.

C. Keep a record of the peak flow meter numbers if symptoms of asthma are getting worse. It is important to keep track of peak flow readings daily and when the patient's symptoms are getting worse. The patient should have specific directions as to when to call the physician based on personal peak flow numbers. Peak flow is measured by exhaling into the meters and should be assessed before and after medications to evaluate their effectiveness.

15. When planning appropriate nursing interventions for a patient with metastatic lung cancer and a 60-pack-year history of cigarette smoking, the nurse recognizes that the smoking has most likely decreased the patient's underlying respiratory defenses because of impairment of which of the following? A. Reflex bronchoconstriction B. Ability to filter particles from the air C. Cough reflex D. Mucociliary clearance

D. Mucociliary clearance Smoking decreases the ciliary action in the tracheobronchial tree, resulting in impaired clearance of respiratory secretions, chronic cough, and frequent respiratory infections.

6. What type of anemia is associated with folate deficiency? A. Microcytic B. Pernicious C. Megaloblastic D. Iron deficiency

C. Megaloblastic Megaloblastic anemia is nutritional anemia; large immature red blood cells with a decreased oxygen-carrying capacity can occur as a result of impaired DNA synthesis. Folic acid is used in the synthesis of DNA and helps convert B12 to coenzyme form. Folic acid is needed for growth and development of red blood cells. Microcytic anemia is anemia with abnormally small erythrocytes (red blood cells) in Hb. This anemia is associated with vitamin B6 (pyridoxine) deficiency. Pernicious anemia is caused by a deficiency of vitamin B12. Iron-deficiency anemia results from loss of blood or deficient intake of iron foods or disease states in which the body does not absorb or utilize iron as it should.

20. During assessment of a 45-year-old patient with asthma, the nurse notes wheezing and dyspnea. The nurse interprets that these symptoms are related to which of the following pathophysiologic changes? A. Laryngospasm B. Overdistention of the alveoli C. Narrowing of the airway D. Pulmonary edema

C. Narrowing of the airwayNarrowing of the airway leads to reduced airflow, making it difficult for the patient to breathe and producing the characteristic wheezing.

16. While ambulating a patient with metastatic lung cancer, the nurse observes a drop in oxygen saturation from 93% to 86%. Which of the following nursing interventions is most appropriate based upon these findings? A. Continue with ambulation as this is a normal response to activity. B. Move the oximetry probe from the finger to the earlobe for more accurate monitoring during activity. C. Obtain a physician's order for supplemental oxygen to be used during ambulation and other activity. D. Obtain a physician's order for arterial blood gas determinations to verify the oxygen saturation.

C. Obtain a physician's order for supplemental oxygen to be used during ambulation and other activity. An oxygen saturation level that drops below 90% with activity indicates that the patient is not tolerating the exercise and needs to have supplemental oxygen applied.

11. When assessing a patient's respiratory status, which of the following nonrespiratory data are most important for the nurse to obtain? A. Height and weight B. Neck circumference C. Occupation and hobbies D. Usual daily fluid intake

C. Occupation and hobbiesMany respiratory problems occur as a result of chronic exposure to inhalation irritants. Common occupational sources of inhalation irritants include mines, granaries, farms, lawn care companies, paint, plastics and rubber manufacture, and building remodeling. Hobbies associated with inhalation irritants include woodworking, metal finishing, furniture refinishing, painting, and ceramics. Daily fluids, height, and weight are more related to respiratory problems secondary to cardiac issues.

30. The nurse determines that a patient is experiencing common adverse effects from the inhaled corticosteroid beclomethasone (Beclovent) after noting which of the following? A. Adrenocortical dysfunction and hyperglycemia B. Elevation of blood glucose and calcium levels C. Oropharyngeal candidiasis and hoarseness D. Hypertension and pulmonary edema

C. Oropharyngeal candidiasis and hoarseness Oropharyngeal candidiasis and hoarseness are common adverse effects from the use of inhaled corticosteroids because the medication can lead to overgrowth of organisms and local irritation if the patient does not rinse the mouth following each dose.

15. When reviewing the results of a 83-year-old patient's blood tests, which of the following findings would be of most concern to the nurse? A. Platelets of 150,000/µl B. Serum iron of 50 mcg/dl C. Partial thromboplastin time (PTT) of 60 seconds D. Erythrocyte sedimentation rate (ESR) of 35 mm in 1 hour

C. Partial thromboplastin time (PTT) of 60 seconds In aging, the partial thromboplastin time (PTT) is normally decreased, so an abnormally high PTT of 60 seconds is an indication that bleeding could readily occur. Platelets are unaffected by aging, and 150,000 is a normal count. Serum iron levels are decreased and the erythrocyte sedimentation rate (ESR) is significantly increased with aging, as are reflected in these values.

4. After a posterior nasal pack is inserted by a physician, the patient is very anxious and states, "I don't feel like I'm breathing right." The immediate intervention the nurse should initiate is to A. monitor ABGs. B. reassure the patient that this is normal discomfort. C. cut the pack strings and pull the packing out with a hemostat. D. direct a flashlight into the patient's mouth and inspect the oral cavity.

D. direct a flashlight into the patient's mouth and inspect the oral cavity. The nurse should inspect the oral cavity for the presence of blood, soft palate necrosis, and proper placement of the posterior plug. If the posterior plug is visible, the physician should be notified for readjustment of the packing. Reassurance, cutting the strings, and ABGs are not top priority interventions. The nurse needs further data before intervening.

10. Anticoagulant therapy is used in the treatment of thromboembolic disease because anticoagulants can A. dissolve the thrombi. B. decrease blood viscosity. C. prevent absorption of vitamin K. D. inhibit the synthesis of clotting factors.

D. inhibit the synthesis of clotting factors. Anticoagulant therapy is based on the premise that the initiation or extension of thrombi can be prevented by inhibiting the synthesis of clotting factors or by accelerating their inactivation. The anticoagulants heparin and warfarin do not induce thrombolysis but effectively prevent clot extension.

7. The most appropriate position to assist a patient with chronic obstructive pulmonary disease (COPD) who is having difficulty breathing would be a A. high Fowler's position without a pillow behind the head. B. semi-Fowler's position with a single pillow behind the head. C. right side-lying position with the head of the bed at 45 degrees' elevation. D. sitting upright and forward position with arms supported on an over-the-bed table.

D. sitting upright and forward position with arms supported on an over-the-bed table.Sitting upright and leaning forward with arms supported on an over-the-bed table would be of most help to this patient, because it allows for expansion of the thoracic cage in all four directions (front, back, and two sides).

15. When administering oxygen to a patient with COPD with the potential for carbon dioxide narcosis, the nurse should A. never administer oxygen at a rate of more than 2 L/min. B. monitor the patient's use of oxygen to detect oxygen dependency. C. monitor the patient for symptoms of oxygen toxicity, such as paresthesias. D. use ABGs as a guide to determine what FIO2 level meets the patient's needs.

D. use ABGs as a guide to determine what FIO2 level meets the patient's needs. It is critical to start oxygen at low flow rates and then use ABGs as a guide to determine what FIO2 level is sufficient and can be tolerated.

gastric lavage

cleansing procedure in which the stomach is irrigated with a prescribed solution

rhonchi

continuous rumbling, snoring, or rattling sounds from obstruction of large airways with secretions; most prominent on expiration; change often evident after coughing or suctioning.

pleural friction rub

creaking or grating sound from roughened, inflamed surfaces of the pleura rubbing together, evident during inspiration, expiration, or both and no change with coughing; usually uncomfortable, especially on deep inspiration.

Cushing syndrome

excess cortisol secretion that doesn't depend on stimulation from Adrenocorticotropic hormone (ACTH) and is associated with disorders of the adrenal glands. The most common of these disorders is a noncancerous tumor of the adrenal cortex, called an adrenal adenoma.

adventitious sounds

extra breath sounds that are not normally heard, such as crackles, rhonchi, wheezes, and pleural friction rubs.

hypercapnia

greater than normal amounts of carbon dioxide in the blood (PaCO2 > 45 mm Hg); also called hypercarbia.

Huntington disease (chorea)

hereditary disorder marked by degenerative changes in the cerebrum leading to abrupt involuntary movements and mental deterioration

cor pulmonale

hypertrophy of the right side of the heart, with or without heart failure, resulting from pulmonary hypertension.

Somogyi effect

hypoglycemia followed by rebound hyperglycemia

Dumping syndrome

increase fat and protein, small frequent meals, lie down after meal to decrease peristalsis. Wait 1 hr after meals to drink

pleurisy (pleuritis)

inflammation of the pleura.

flail chest

instability of the chest wall resulting from multiple rib fractures.

climacteric

midlife transition in which fertility declines

normocapnia

normal arterial carbon dioxide pressure (PaCO2 35 to 45 mm Hg).

epistaxis

nosebleed

placental abruption

premature separation of the placenta

vibration

pressing on the chest with the flat of the hands while repeatedly tensing the hand and arm muscles to facilitate movement of secretions to larger airways.

hiatal hernia

protrusion of a part of the stomach upward through the opening in the diaphragm

chronic pancreatitis

pulmonary disease state characterized by the presence of airflow obstruction caused by chronic bronchitis or emphysema; clinical use of the term indicates the presence of chronic bronchitis and/or emphysema; includes asthma, chronic bronchiectasis, chronic bronchitis, and emphysema.

mechanical receptors

receptors located in lungs, upper airways, chest wall, and diaphragm that are stimulated by irritants, muscle stretching, and alveolar wall distortion.

Primordial prevention care

refers to prevention of the risk factors themselves at either the social or environmental level.

chest percussion

rhythmic percussion of a patient's chest with cupped hands to loosen retained respiratory secretions.

Primary Preventive Care

routine medical care and screening generally provided by physicians specializing in family practice, general internal medicine, and pediatrics

Tetralogy of Fallot (TOF)

set of four congenital heart defects occurring together

crackle

short, low-pitched sounds consisting of discontinuous bubbling caused by air passing through airway intermittently occluded by mucus, unstable bronchial wall, or fold of mucosa; evident on inspiration and, at times, expiration; similar sound to blowing through a straw under water.

dyspnea

shortness of breath; difficulty breathing that may be caused by certain heart conditions, strenuous exercise, or anxiety.

tracheostomy

surgical opening into the trachea through which an indwelling tube may be inserted.

paracentesis

surgical puncture to remove fluid from the abdomen

allergic rhinitis

the reaction of the nasal mucosa to a specific allergen.

rhinoplasty

the surgical reconstruction of the nose.

elastic recoil

the tendency for the lungs to recoil or reduce in volume after being stretched or expanded.

postural drainage

the use of various positions to promote gravity drainage of bronchial secretions; coughing usually expels secretions of the trachea.

centrilobular emphysema

type of emphysema often associated with chronic bronchitis in which respiratory bronchioles enlarge, the walls are destroyed, and the bronchioles become confluent; characterized by enlargement of air spaces in the proximal part of the acinus, primarily at the level of the respiratory bronchioles.

fremitus

vibration of the chest wall produced by vocalization.

31. A patient's ABGs include a PaO2 of 88 mm Hg and a PaCO2 of 38 mm Hg and mixed venous blood gases include a PvO2 of 40 mm Hg and PvCO2 of 46 mm Hg. These findings indicate that the patient has A. impaired cardiac output. B. unstable hemodynamics. C. inadequate delivery of oxygen to the tissues. D. normal capillary oxygen-carbon dioxide exchange.

D. normal capillary oxygen-carbon dioxide exchange. Normal venous blood gas values reflect the normal uptake of oxygen from arterial blood and the release of carbon dioxide from cells into the blood, resulting in a much lower PaO2 and an increased PaCO2. The pH is also decreased in mixed venous blood gases because of the higher PvCO2. Normal mixed venous blood gases also have much lower PvO2 and SvO2 than arterial blood bases. Mixed venous blood gases are used when patients are hemodynamically unstable to evaluate the amount of oxygen delivered to the tissue and the amount of oxygen consumed by the tissues.

Titrate

continuously measure and adjust the balance of (a physiological function or drug dosage).

cystic fibrosis

an autosomal recessive, multisystem disease characterized by altered function of the exocrine glands involving primarily the lungs, pancreas, and sweat glands.

tidal volume

volume of air exchanged with each breath.

SBAR communication

(Situation, Background, Assessment, Recommendation) - framework for communication between members of the healthcare team about a patient's condition.

When the cervix of a woman in labor is dilated 9 cm, she states that she has the urge to push. Which action should the nurse implement at this time? 1 Having her pant-blow during contractions 2 Placing her legs in stirrups to facilitate pushing 3 Encouraging her to bear down with each contraction 4 Reviewing the pushing techniques taught in childbirth classes

1

chemoreceptor

a sensory nerve cell that responds to a change in the chemical composition (PaCO2 and pH) of the fluid around it.

chest physiotherapy

a series of maneuvers including percussion, vibration, and postural drainage designed to promote clearance of excessive respiratory secretions.

41. Select all that apply. During initial assessment, a nurse should record which of the following manifestations of respiratory distress? A. Tachypnea B. Nasal flaring C. Thready pulse D. Panting or grunting E. Use of intercostal muscles F. An inspiratory-to-expiratory ratio of 1:2

AD Manifestations of respiratory distress include tachypnea, grunting and panting on respiration, central cyanosis, use of accessory muscles, and flaring nares.

preeclampsia

abnormal condition associated with pregnancy, marked by high blood pressure, proteinuria, edema, and headache

dry powder inhaler

dry powdered drug delivered by inhalation.

thoracotomy

surgical opening into the thoracic cavity.

decompensated shock

when the body can no longer compensate for low blood volume or lack of perfusion. Late signs such as decreasing blood pressure become evident

Dependent edema usually occurs

with severe protein deficiency and heart failure. Spoon-shaped nails usually occur with iron deficiency anemia

multigravida

woman who has been pregnant more than once

Which hormone elevations indicate Turner syndrome? Select all that apply. 1 Lutropin 2 Prolactin 3 Follitropin 4 Testosterone 5 Progesterone

1, 3 Elevation of lutropin and follitropin indicates Turner syndrome. Elevation of prolactin indicates possible galactorrhea, pituitary tumor, disease of hypothalamus or pituitary gland, and hypothyroidism. Elevated testosterone levels in women indicate adrenal neoplasm, ovarian neoplasm, and polycystic ovary syndrome. Elevated progesterone levels in men indicate possible testicular tumors and hyperthyroidism. Elevated progesterone levels in women indicate possible ovarian luteal cysts.

A client who menstruates regularly every 30 days asks a nurse on what day she is most likely to ovulate. Because the client's last menses started on January 1, the nurse should tell her that ovulation should occur on which day in January? 1 7 2 16 3 24 4 29

2 Ovulation should occur on January 16. The time between ovulation and the next menstruation is relatively constant. In a 30-day cycle the first 15 days are preovulatory, ovulation occurs on day 16, and the next 14 days are postovulatory. January 7, January 24, and January 29 all reflect inaccurate calculation of the date of ovulation.

When an intestinal obstruction is suspected, a client has a nasogastric tube inserted and attached to suction. What response should the nurse critically assess on this client? 1 Edema 2 Belching 3 Fluid deficit 4 Excessive salivation

3 Dehydration is a danger because of fluid loss with gastrointestinal (GI) suction. Based on the data provided, edema, belching, and excessive salivation are not likely to occur.

What should supportive nursing care at the beginning of the mother-infant relationship include? 1 Suggesting that the mother choose breastfeeding instead of formula feeding 2 Advising the mother to engage in rooming-in with the newborn at the bedside 3 Encouraging the mother to help out with simple aspects of her newborn's care 4 Observing the mother-infant interaction unobtrusively to evaluate the relationship

3 Holding, touching, and interacting with the newborn while providing basic care promotes attachment. The nurse's infant feeding preference should not be forced upon the mother. Although rooming-in helps promote attachment, not all women have the physical or emotional ability to provide 24-hour care to the newborn so early in the postpartum period. Early observation is not adequate; full evaluation of the relationship can be achieved only by allowing the mother ample time to interact with her baby.

Which cartilage is also known as the Adam's apple? 1 Costal 2 Cricoid 3 Thyroid 4 Arytenoid

3 The thyroid cartilage is commonly known as the "Adam's apple" and is the largest of the cartilages that comprise the laryngeal skeleton. The costal cartilage is only found at the anterior ends of the rib. The cricoid cartilage lies below the thyroid cartilage. The arytenoid cartilage works with the thyroid cartilage in facilitating vocal cord movement.

What statement by a breast-feeding mother indicates that the nurse's teaching regarding stimulating the let-down reflex has been successful? 1 "I will take a cool shower before each feeding." 2 "I will drink a couple of quarts of fat-free milk a day." 3 "I will wear a snug-fitting breast binder day and night." 4 "I will apply warm packs and massage my breasts before each feeding."

4 Applying warm packs and massaging the breasts before each feeding help dilate milk ducts, promote emptying of the breasts, and stimulate further lactation. Taking a cool shower before each feeding will contract the milk ducts and interfere with the let-down reflex. Heavy consumption of milk products is not required to stimulate the production of milk. Breast binders may inhibit lactation by fooling the body into thinking that milk secretion is no longer needed.

What are the cardiovascular manifestations observed in a client with adrenal insufficiency? 1 Fatigue 2 Salt craving 3 Weight loss 4 Hyponatremia

4 Hyponatremia is a decrease in serum sodium levels, which is the cardiovascular manifestation of adrenal insufficiency. Fatigue is a neuromuscular manifestation observed in clients with adrenal insufficiency, while salt cravings and weight loss are the abdominal manifestations observed in clients with adrenal insufficiency.

A client undergoing presurgical testing before a total abdominal hysterectomy says to the nurse, "After I have this surgery I know my husband will never come near me again." What is the nurse's best initial response? 1 "You're underestimating his love for you." 2 "You're wondering about the effect on your sexual relations." 3 "You're worried that the surgery will change how others see you." 4 "You're concerned about how your husband will respond to your surgery."

4 Stating that the client is concerned about how her husband will respond to her surgery is an open-ended response that encourages further discussion without focusing on an area that the nurse, not the client, feels is the problem. Accusing the client of underestimating her husband's love denies the client's feeling and may cause feelings of guilt for questioning the partner's love. Wondering about the effect on sexual relations is too specific; the nurse does not have enough information to come to this conclusion. Worrying that the surgery will change how others see the client shifts the focus from the client's voiced concerns; the client specifically referred to her husband, not others.

incomplete abortion

Expulsion of the fetus with retained placenta before 20 weeks' gestation.

Factor V Leiden

Most common hypercoagulable state, mutated factor V that is resistant to , Protein C and Protein S cleavage.

gerontological

Pertaining to the study of the non-medical problems of the aged

Addison Disease S/S

Steroid Abuse/Chronic use Hypovolemia/glycemia/natremia Loss of Body Hair Hyperpigmentation

Acculturation

The adoption of cultural traits, such as language, by one group under the influence of another.

Factor V Leiden mutation

The most common inherited cause of hypercoagulability.

pneumothorax

a collection of air or gas in the pleural space causing the lung to collapse.

metered-dose inhaler

aerosolized drug delivered in a specific amount by activating the inhaler or by inhaling.

hypocapnia

low arterial carbon dioxide pressure (PaCO2 < 35 mm Hg); also called hypocarbia.

chronic bronchitis

obstructive pulmonary disease characterized by excessive production of mucus and chronic inflammatory changes in the bronchi, resulting in a cough with expectoration for at least 3 months of the year for more than 2 consecutive years.

hospital-acquired pneumonia

pneumonia occurring 48 hours or longer after hospital admission and not incubating at the time of hospitalization.

wheezes

a form of rhonchus characterized by continuous high-pitched squeaking sound caused by rapid vibration of bronchial walls.

A nurse is advising a client about the risks associated with failing to seek treatment for acute pharyngitis caused by beta-hemolytic streptococcus. For what health problem is the client at risk? 1 Asthma 2 Anemia 3 Endocarditis 4 Reye syndrome

3 Streptococcal infection can be spread through the circulation to the heart; endocarditis results and affects the valves of the heart. Asthma, anemia, and Reye syndrome are not caused by beta-hemolytic streptococcus.

One hour after a birth a nurse palpates a client's fundus to determine whether involution is taking place. The fundus is firm, in the midline, and two fingerbreadths below the umbilicus. What should the nurse do next? 1 Encourage the client to void. 2 Notify the practitioner immediately. 3 Massage the uterus and attempt to express clots. 4 Continue periodic assessments and record the findings

4 Immediately after birth the uterus is 2 cm below the umbilicus; during the first several postpartum hours the uterus will rise slowly to just above the level of the umbilicus. These findings are expected, and they should be recorded. Encouraging the client to void is unnecessary; if the bladder is full, the uterus will be higher and pushed to one side. Notifying the healthcare provider is unnecessary; involution is occurring as expected. Massage is used when the uterus is soft and "boggy."

Turner Syndrome

A chromosomal disorder in females in which either an X chromosome is missing, making the person XO instead of XX, or part of one X chromosome is deleted.

amniocentesis

A technique of prenatal diagnosis in which amniotic fluid, obtained by aspiration from a needle inserted into the uterus, is analyzed to detect certain genetic and congenital defects in the fetus.

A client is diagnosed with an acute small bowel obstruction. Which assessment finding requires the most immediate intervention by the nurse? A.Fever of 102° F B.Blood pressure of 150/90 mm Hg C.Abdominal cramping D.Dry mucous membranes

A A sudden increase in temperature is an indicator of peritonitis. The nurse should notify the health care provider immediately (A). (B, C, and D) are also findings that require intervention by the nurse, but are of less priority than (A). (B) may indicate a hypertensive condition but is not as acute a condition as peritonitis. (C) is an expected finding in clients with small bowel obstruction and may require medication. (D) indicates probable fluid volume deficit, which requires fluid volume replacement.

The nurse is administering a nystatin suspension (Mycostatin) for stomatitis. Which instruction will the nurse provide to the client when administering this medication? A."Hold the medication in your mouth for a few minutes before swallowing it." B."Do not drink or eat milk products for 1 hour prior to taking this medication." C."Dilute the medication with juice to reduce the unpleasant taste and odor." D."Take the medication before meals to promote increased absorption."

A Mycostatin is prescribed for fungal infections of the mouth. The client should swish the medication in the mouth for 2 minutes and then swallow (A). (B) does not affect administration of this medication. The medication should not be diluted because this will reduce its effectiveness (C). (D) is not necessary.

A 63-year-old client with type 2 diabetes mellitus is admitted for treatment of an ulcer on the heel of the left foot that has not healed with wound care. The nurse observes that the entire left foot is darker in color than the right foot. Which additional symptom should the nurse expect to find? A.Pedal pulses will be weak or absent in the left foot. B.The client will state that the left foot is usually warm. C.Flexion and extension of the left foot will be limited. D.Capillary refill of the client's left toes will be brisk.

A Symptoms associated with decreased blood supply are weak or absent pedal and tibial pulses (A). The client with diabetes experiences vascular scarring as a result of atherosclerotic changes in the peripheral vessels. This results in compromised perfusion to the dependent extremities, which further delays wound healing in the affected foot. Although flexion and extension may be limited (C), depending on the degree of damage, this is not always the case. (B and D) are signs of adequate perfusion of the foot, which would not be expected in this client.

surfactant

a lipoprotein that lowers the surface tension in the alveoli, reduces the amount of pressure needed to inflate the alveoli, and decreases the tendency of the alveoli to collapse.

A family member was taught to suction a client's tracheostomy prior to the client's discharge from the hospital. Which observation by the nurse indicates that the family member is capable of correctly performing the suctioning technique? A.Turns on the continuous wall suction to −190 mm Hg B.Inserts the catheter until resistance or coughing occurs C.Withdraws the catheter while maintaining suctioning D.Reclears the tracheostomy after suctioning the mouth

B (B) indicates correct technique for performing suctioning. Suction pressure should be between −80 and −120 mm Hg, not −190 mm Hg (A). The catheter should be withdrawn 1 to 2 cm at a time with intermittent, not continuous, suction (C). (D) introduces pathogens unnecessarily into the tracheobronchial tree.

The nurse is assessing a male client with acute pancreatitis. Which finding requires the MOST immediate intervention by the nurse? A.The client's amylase level is three times higher than the normal level. B.While the nurse is taking the client's blood pressure, he has a carpal spasm. C.On a 1 to 10 scale, the client tells the nurse that his epigastric pain is at 7. D.The client states that he will continue to drink alcohol after going home.

B A positive Trousseau sign (B) indicates hypocalcemia and always requires further assessment and intervention, regardless of the cause (40% to 75% of those with acute pancreatitis experience hypocalcemia, which can have serious, systemic effects). A key diagnostic finding of pancreatitis is serum amylase and lipase levels that are two to five times higher than the normal value (A). Severe boring pain is an expected symptom for this diagnosis (C), but dealing with the hypocalcemia is a priority over administering an analgesic. Long-term planning and teaching (D) do not have the same immediate importance as a positive Trousseau sign.

community-acquired pneumonia

a lower respiratory tract infection of the lung parenchyma with onset in the community or during the first 2 days of hospitalization.

compliance

a measure of the ease of expansion of the lungs and thorax.

What is the most important nursing priority for a client who has been admitted for a possible kidney stone? A.Reducing dairy products in the diet B.Straining all urine C.Measuring intake and output D.Increasing fluid intake

B Straining all urine (B) is the most important nursing action to take in this case. Encouraging fluid intake (D) is important for any client who may have a kidney stone, but is even more important to strain all urine. Straining urine will enable the nurse to determine when the kidney stone has been passed and may prevent the need for surgery. (C) is not the highest priority action. (A) is usually not recommended until the stone is obtained and the content of the stone is determined. Even then, dietary restrictions are controversial.

The nurse notes that a client who is scheduled for surgery the next morning has an elevated blood urea nitrogen (BUN) level. Which condition is most likely to have contributed to this finding? A.Myocardial infarction 2 months ago B.Anorexia and vomiting for the past 2 days C.Recently diagnosed type 2 diabetes mellitus D.Skeletal traction for a right hip fracture

B The blood urea nitrogen (BUN) level indicates the effectiveness of the kidneys in filtering waste from the blood. Dehydration, which could be caused by vomiting, would cause an increased the BUN level (B). (A) would affect serum enzyme levels, not the BUN level. (C) would primarily affect the blood glucose level; renal failure that could increase the BUN level would be unlikely in a client newly diagnosed with type 2 diabetes. Effects of (D) might affect the complete blood count (CBC) but would not directly increase the BUN level.

The nurse teaches a client with type 2 diabetes nutritional strategies to decrease obesity. Which food item(s) chosen by the client INDICATES UNDERSTANDING of the teaching? (Select all that apply.) A.White bread B.Salmon C.Broccoli D.Whole milk E.Banana

B, C, E (B, C, and E) provide fresh fruits, lean meats and fish, vegetables, whole grains, and low-fat dairy products. All are recommended by the American Diabetes Association (ADA) and are a part of the My Plate guidelines recommended by the U.S. Department of Agriculture (USDA). Whole milk (D) is high in fat and is not recommended by ADA. White bread is milled, a process that removes the essential nutrients. It should be avoided for weight loss and is a poor choice for the client with diabetes (A).

53. A patient is being discharged from the emergency department after being treated for epistaxis. In teaching the family first aid measures in the event the epistaxis would recur, which of the following measures would the nurse suggest? (Select all that apply.) A. Tilt patients head backwards B. Apply ice compresses to the nose C. Pinch the entire soft lower portion of the nose D. Partially insert a small gauze pad into the bleeding nostril

B,C,D First aid measures to control epistaxis includes placing the patient in a sitting position, leaning forward. Tilting the head back does not stop the bleeding, but rather allows the blood to enter the nasopharynx, which could result in aspiration or nausea/vomiting from swallowing blood. All of the other options are appropriate first aid treatment of epistaxis.

esophageal speech

a method of swallowing air, trapping it in the esophagus, and releasing it to create sound.

40. The patient has an order for albuterol 5 mg via nebulizer. Available is a solution containing 2 mg/ml. How many milliliters should the nurse use to prepare the patient's dose? A. 0.2 B. 2.5 C. 3.75 D. 5.0

B. 2.5

Raynaud's disease

a peripheral arterial occlusive disease in which intermittent attacks are triggered by cold or stress

A client is ready for discharge following the creation of an ileostomy. Which instruction should the nurse include in discharge teaching? A.Replace the stoma appliance every day. B.Use warm tap water to irrigate the ileostomy. C.Change the bag when the seal is broken. D.Measure and record the ileostomy output.

C A seal must be maintained to prevent leakage of irritating liquid stool onto the skin (C). (A) is excessive and can cause skin irritation and breakdown. Ileostomies produce liquid fecal drainage, so (B) is not necessary. (D) is not needed.

A 62-year-old woman who lives alone tripped on a rug in her home and fractured her hip. Which predisposing factor most likely contributed to the fracture in the proximal end of her femur? A.Failing eyesight resulting in an unsafe environment B.Renal osteodystrophy resulting from chronic kidney disease (CKD) C.Osteoporosis resulting from declining hormone levels D.Cerebral vessel changes causing transient ischemic attacks

C The most common cause of a fractured hip in older women is osteoporosis, resulting from reduced calcium in the bones as a result of hormonal changes in the perimenopausal years (C). (A) may or may not have contributed to the accident, but eye changes were not involved in promoting the hip fracture. (B) is not a common condition of older people but is associated with CKD. Although (D) may result in transient ischemic attacks (TIAs) or stroke, it will not result in fragility of the bones, as does osteoporosis.

The nurse includes frequent oral care in the plan of care for a client scheduled for an esophagogastrostomy for esophageal cancer. This intervention is included in the client's plan of care to address which nursing diagnosis? A.Fluid volume deficit B.Self-care deficit C.Risk for infection D.Impaired nutrition

C The primary reason for performing frequent mouth care preoperatively is to reduce the risk of postoperative infection (C) because these clients may be regurgitating retained food particles, blood, or pus from the tumor. Meticulous oral care should be provided several times a day before surgery. Although oral care will be of benefit to the client who may also be experiencing (A, B, or D), these problems are not the primary reason for the provision of frequent oral care.

A client is being discharged following radioactive seed implantation for prostate cancer. What is the most important information that the nurse should provide to this client's family? A.Follow exposure precautions. B.Encourage regular meals. C.Collect all urine. D.Avoid touching the client.

Clients being treated for prostate cancer with radioactive seed implants should be instructed regarding the amount of time and distance needed to prevent excessive exposure (A) that would pose a hazard to others. (B) is a good suggestion to promote adequate nutrition but is not as important as (A). (C) is unnecessary. Contact with the client (D) IS permitted but should be BRIEF to limit radiation exposure.

pneumonia

an acute inflammation of the lungs, often caused by inhaled pneumococci of the species Streptococcus pneumoniae.

The nurse assesses a client who has been prescribed furosemide (Lasix) for cardiac disease. Which electrocardiographic change would be a concern for a client taking a diuretic? A.Tall, spiked T waves B.A prolonged QT interval C.A widening QRS complex D.Presence of a U wave

D A U wave (D) is a positive deflection following the T wave and is often present with hypokalemia (low potassium level). (A, B, and C) are all signs of hyperkalemia.

The nurse is preparing a 45-year-old client for discharge from a cancer center following ileostomy surgery for colon cancer. Which discharge goal should the nurse include in this client's discharge plan? A.Reduce the daily intake of animal fat to 10% of the diet within 6 weeks. B.Exhibit regular, soft-formed stool within 1 month. C.Demonstrate the irrigation procedure correctly within 1 week. D.Attend an ostomy support group within 2 weeks.

D Attending a support group (D) will be beneficial to the client and should be encouraged because adaptation to the ostomy can be difficult. This goal is attainable and is measurable. (A) is not specifically related to ileostomy care. The client with an ileostomy will not be able to accomplish (B). (C) is not necessary.

A female client who received a nephrotoxic drug is admitted with acute renal failure and asks the nurse if she will need dialysis for the rest of her life. Which pathophysiologic consequence should the nurse explain that supports the need for temporary dialysis until acute tubular necrosis subsides? A.Azotemia B.Oliguria C.Hyperkalemia D.Nephron obstruction

D CKD is characterized by progressive and irreversible destruction of nephrons, frequently caused by hypertension and diabetes mellitus. Nephrotoxins cause acute tubular necrosis, a reversible acute renal failure, which creates renal tubular obstruction from endothelial cells that are sloughed or become edematous. The obstruction of urine flow will resolve (D) with the return of an adequate glomerular filtration rate and, when it does, dialysis will no longer be needed. (A, B, and C) are manifestations seen in the acute and chronic forms of kidney disease.

39. The nurse is scheduled to give a dose of ipratropium bromide by metered dose inhaler. The nurse would administer the right drug by selecting the inhaler with which of the following trade names? A. Vanceril B. Pulmicort C. AeroBid D. Atrovent

D. Atrovent The trade or brand name for ipratropium bromide, an anticholinergic medication, is Atrovent.

knee gatch

Elevating this can put pressure on the popliteal space, restrict circulation and increase the risk of thrombophlebitis.

O2 toxicity

a condition of oxygen overdosage caused by prolonged exposure to a high levels of oxygen; may inactivate pulmonary surfactant and lead to development of acute respiratory distress syndrome.

deviated septum

a deflection of the normally straight nasal septum.

asthma

a chronic inflammatory lung disease that results in airflow obstruction; characterized by recurring episodes of paroxysmal dyspnea, wheezing on expiration and/or inspiration caused by constriction of the bronchi, coughing, and viscous mucoid bronchial secretions.

pancreatic insufficiency

a condition characterized by inadequate production and secretion of pancreatic hormones or enzymes.

tension pneumothorax

a pneumothorax with rapid accumulation of air in the pleural space causing severely high intrapleural pressures with resultant tension on the heart and great vessels.

tuberculosis

an infectious disease caused by Mycobacterium tuberculosis; usually involves the lungs but also occurs in the larynx, kidneys, bones, adrenal glands, lymph nodes, and meninges and can be disseminated throughout the body.

panlobular emphysema

type of emphysema involving distention and destruction of the entire primary respiratory lobule; usually associated with "1-antitrypsin deficiency; also called chronic hypertrophic, diffuse, generalized, panlobular, or vesicular emphysema.

pulmonary hypertension

elevated pulmonary pressure resulting from an increase in pulmonary vascular resistance to blood flow through small arteries and arterioles.

hyperresponsiveness

excessive or exaggerated response to a stimulus; in asthma leads to bronchoconstriction in response to physical, chemical, or pharmacologic stimuli.

ascites

abnormal accumulation of fluid in the abdomen

preclampsia

abnormal condition encountered during pregnancy or shortly after delivery characterized by high blood pressure, edema, and proteinuria, but with no convulsions or coma

amenorrhea

absence of menstruation

hemothorax

accumulation of blood in the pleural space.

cardiac tamponade

acute compression of the heart caused by fluid accumulation in the pericardial cavity

absorption atelectasis

alveolar collapse that occurs when high concentrations of oxygen are given and oxygen replaces nitrogen in the alveoli; if airway obstruction occurs, the oxygen is absorbed into the bloodstream and the alveoli collapse.

pulmonary edema

an abnormal accumulation of fluid in the alveoli and interstitial spaces of the lungs caused most commonly by heart failure; an acute, life-threatening situation in which the lung alveoli become filled with serous or serosanguineous fluid caused most commonly by heart failure.

pleural effusion

an abnormal accumulation of fluid in the intrapleural spaces of the lungs.

hyperreactivity

an abnormal condition in which responses to stimuli are exaggerated.

emphysema

an abnormal condition of the pulmonary system, characterized by overinflation and destructive changes in alveolar walls.

apnea

an absence of spontaneous respirations.

empyema

an accumulation of purulent exudates in a body cavity, especially the pleural space, as a result of bacterial infection, such as pleurisy or tuberculosis.

acute bronchitis

an inflammation of the lower respiratory tract that is usually due to infection.

Frequency of contractions

beginning of one contraction to the beginning of the next

nasal polyps

benign mucous membrane masses that form slowly in response to repeated inflammation of the sinus or nasal mucosa and project into the nasal cavity.

diaphragmatic breathing

breathing with the use of the diaphragm to achieve maximum inhalation and slow respiratory rate.

pulmonary capillary wedge pressure (PCWP)

- indicates Left ventricular failure. - Fluid overload.

Tertiary preventive care

-Goal is to stop disease progression; return to pre-illness state -Medications, Surgical Treatment, Physical Therapy/Rehabilitation

A client admitted to the high-risk unit with a threatened abortion anxiously asks the nurse, "Could this have happened because I had the flu?" How should the nurse respond? 1 "Tell me why you feel this way. Do you think that you did something to cause the bleeding?" 2 "We know that maternal infection sometimes results in spontaneous abortion. Perhaps the flu did cause it." 3 "I'm sure that there's nothing you could have done to cause this. You shouldn't worry about it." 4 "The primary healthcare provider will be here soon and will be better prepared to answer your questions. Why don't you wait until then?"

1 Asking the client to talk about how she feels encourages the client to discuss her fears and anxieties. Stating that the flu may have caused the spontaneous abortion gives inaccurate information; this conclusion has not been documented, and this response adds to the guilt felt by the client. Telling the client that there is nothing she could have done to cause the problem does not focus on the client's feelings; it cuts off communication between the nurse and the client. Telling the client to wait until the primary healthcare provider arrives denies the client's feelings, abdicates the nurse's responsibility to the client, and cuts off communication. Also, it may increase anxiety because it implies that the nurse is not adequately prepared to care for the client.

A nurse is caring for a client with Addison disease. Which dietary instruction should the nurse teach the client to follow? 1 Add extra salt to food 2 Consume high-potassium foods 3 Omit protein foods at each meal 4 Restrict the daily intake of fluids to 1 L

1 Because of diminished mineralocorticoid secretion, clients with Addison disease are prone to developing hyponatremia. Therefore, the addition of salt to the diet is advised. Clients with Addison disease are prone to hyperkalemia. High-potassium foods can be restricted. Protein is not omitted from the diet; ingestion of essential amino acids is necessary for optimum metabolism and healing. Fluids are not restricted for clients with Addison disease.

A client in labor begins to experience contractions 2 to 3 minutes apart and lasting about 45 seconds. Between contractions the nurse identifies a fetal heart rate (FHR) of 100 beats/min on the internal fetal monitor. What is the priority nursing action? 1 Notifying the healthcare provider 2 Resuming continuous fetal heart monitoring 3 Continuing to monitor the maternal vital signs 4 Documenting the fetal heart rate as an expected response to contractions

1 Bradycardia (baseline FHR slower than 110 beats/min) indicates that the fetus may be compromised, requiring medical intervention. Resuming continuous fetal heart monitoring may be dangerous. The fetus may be compromised, and time should not be spent on monitoring. Continuing to monitor the maternal vital signs is not the priority at this time. The expected FHR is 110 to 160 beats/min between contractions.

A client has surgery for the creation of a colostomy. Postoperatively, what color does the nurse expect a viable stoma to be? 1 Brick red 2 Pale pink 3 Light gray 4 Dark purple

1 Brick red describes a stoma that has adequate vascular perfusion. Pale pink indicates inadequate perfusion of the stoma. Light gray is indicative of poor tissue perfusion. Dark purple indicates inadequate perfusion of the stoma.

What is the priority nursing intervention during the admission of a primigravida in labor? 1 Monitoring the fetal heart rate 2 Asking the client when she ate last 3 Obtaining the client's health history 4 Determining whether the membranes have ruptured

1 Determining fetal well-being supersedes all other measures; if the fetal heart rate is absent or persistently decelerating, immediate intervention is required. The health history, including the client's last meal and whether the membranes have ruptured, may be taken once fetal well-being has been established.

A client is in the intensive care unit. The nurse observing the telemetry monitor identifies flattening T waves and peaked P waves. What problem should the nurse consider based on these ECG changes? 1 Hypokalemia 2 Hypocalcemia 3 Hyponatremia 4 Hypomagnesemia

1 Flattened or inverted T waves, peaked P waves, depressed ST segments, and elevated U waves are associated with hypokalemia. Prolongation of the QT interval may indicate hypocalcemia. Hyponatremia is not reflected in the heart's electrical conduction. Although flattening of T waves may occur with hypomagnesemia, the ST segment may be shortened, and the PR and QRS intervals may be prolonged.

The nurse is caring for a client in her third trimester who is scheduled for an amniocentesis. What should the nurse do to prepare the client for this test? 1 Instruct her to void immediately before the test. 2 Tell her to assume the high Fowler position before the test. 3 Encourage her to drink three glasses of water before the test. 4 Advise her to take nothing by mouth for several hours before the test.

1 The client is instructed to void immediately before the test to help prevent injury to the bladder as the needle is introduced into the amniotic sac. The supine position with a hip roll under the right hip is the preferred position for this procedure. Telling the client to assume the high Fowler position before the test will cause the bladder to fill, making it vulnerable to injury as the needle is inserted into the amniotic sac. Encouraging the client to drink three glasses of water before the test is advised if the amniocentesis is being performed early during pregnancy. There is no reason to withhold food or fluid, because the test does not involve the gastrointestinal tract.

During the assessment of a client in labor, the cervix is determined to be dilated 4 cm. What stage of labor does the nurse record? 1 First 2 Second 3 Prodromal 4 Transitional

1 The first stage of labor is from zero cervical dilation to full cervical dilation (10 cm). The second stage is from full cervical dilation to delivery. The prodromal stage is before cervical dilation begins. The transitional phase is the first stage of labor, from 8 cm of dilation to 10 cm of dilation.

In a noisy room a sleeping newborn initially startles and exhibits rapid movements; however, the baby soon goes back to sleep. What is the most appropriate nursing action in response to this behavior? 1 Documenting an intact reflex 2 Assessing the infant's vital signs 3 Testing the infant's ability to hear 4 Stimulating the infant's respirations

1 The initial response is a reflection of the startle reflex; when the stimulus is repetitive, the response to the stimulus decreases. This decrease in response is called habituation and is expected. Assessing the infant's vital signs and stimulating the infant's respirations are not necessary because the neonate's response is expected. The infant is responding to noise and therefore hears.

A client's arterial blood gas report indicates that pH is 7.25, Pco2 is 60 mm Hg, and HCO3 is 26 mEq/L (26 mmol/L). Which client should the nurse consider is most likely to exhibit these blood gas results? 1 A 65-year-old with pulmonary fibrosis 2 A 24-year-old with uncontrolled type 1 diabetes 3 A 45-year-old who has been vomiting for 3 days 4 A 54-year-old who takes sodium bicarbonate for indigestion

1 The low pH and elevated Pco2 are consistent with respiratory acidosis, which can be caused by pulmonary fibrosis, which impedes the exchange of oxygen and carbon dioxide in the lung. A 24-year-old with uncontrolled type 1 diabetes most likely will experience metabolic acidosis from excess ketone bodies in the blood. A 45-year-old who has been vomiting for 3 days most likely will experience metabolic alkalosis from the loss of hydrochloric acid from vomiting. A 54-year-old who takes sodium bicarbonate for indigestion most likely will experience metabolic alkalosis from an excess of base bicarbonate.

Which statements regarding the involution process are correct? Select all that apply. 1 Involution begins immediately after expulsion of the placenta. 2 Involution is the self-destruction of excess hypertrophied tissue. 3 Involution progresses rapidly during the next few days after birth. 4 Involution is the return of the uterus to a nonpregnant state after birth. 5 Involution may be caused by retained placental fragments and infections.

1,3,4 The involution process is the return of the uterus to a nonpregnant state after birth; it begins immediately after expulsion of the placenta and contraction of the uterine smooth muscle. This process progresses rapidly during the first few days after birth. Subinvolution is the self-destruction of excess hypertrophied tissue; this process may be caused by retained placental fragments or infection.

The nurse is counseling a woman who has just been identified as having a multiple gestation. Why does the nurse consider this pregnancy high risk? 1 Postpartum hemorrhage is an expected complication. 2 Perinatal mortality is two to three times more likely in multiple than in single births. 3 Optimal psychological adjustment after a multiple birth requires 6 months to 1 year. 4 Maternal mortality is higher during the prenatal period in the setting of multiple gestation

2 Perinatal morbidity and mortality rates are higher with multiple-gestation pregnancies, because the greater metabolic demands and the possibility of malpositioning of one or more fetuses increases the risk for complications. Although postpartum hemorrhage does occur more frequently after multiple births, it is not an expected occurrence. Adjustment to a multiple gestation and birth is individual; the time needed for adjustment does not place the pregnancy at high risk. Maternal mortality during the prenatal period is not increased in the presence of a multiple gestation.

A primary healthcare provider prescribes an antihypertensive medication. Which over-the-counter medication should the nurse teach the client to avoid because it has the potential to counteract the effect of the antihypertensive? 1 Omeprazole 2 Acetaminophen 3 Docusate sodium 4 Pseudoephedrine

2 Pseudoephedrine has a pressor effect that may counteract antihypertensive medications, causing an increase in blood pressure. Omeprazole does not interact with antihypertensives. However, it can increase the action of phenytoin, digoxin, clopidogrel, and cyclosporine. Acetaminophen does not have to be avoided when receiving an antihypertensive. Docusate sodium does not have to be avoided when receiving an antihypertensive.

A nurse on the high-risk unit is caring for a client with severe preeclampsia. Which intervention is the most effective in preventing a seizure? 1 Providing a plastic airway 2 Controlling external stimuli 3 Having emergency equipment available 4 Keeping calcium gluconate at the bedside

2 Reducing lights, noise, and stimulation minimizes central nervous system irritability, which can trigger a seizure. A plastic airway will not prevent a seizure. Available emergency equipment will not prevent a seizure, although oxygen and suction equipment may be useful after a seizure. Calcium gluconate is the antidote for magnesium sulfate toxicity; it does not prevent seizures.

The nurse admits a client with preeclampsia to the high-risk prenatal unit. What is the next nursing action after the vital signs have been obtained? 1 Calling the primary healthcare provider 2 Checking the client's reflexes 3 Determining the client's blood type 4 Administering the prescribed intravenous (IV) normal saline

2 The client is exhibiting signs of preeclampsia. The presence of hyperreflexia indicates central nervous system irritability, a sign of a worsening condition. Checking the client's reflexes will help direct the primary healthcare provider to appropriate interventions and alert the nurse to the possibility of seizures. Although the primary healthcare provider will be called, a complete assessment should be performed first to obtain the information needed. Determining the client's blood type is not necessary at this time; assessment of neurologic status is the priority. An IV may be started after the assessment; however, a more dilute saline solution will be prescribed.

Thick mucous gland secretions, elevated sweat electrolytes, meconium ileus, and difficulty maintaining and gaining weight are associated with which autosomal recessive disorder? 1 Cerebral palsy 2 Cystic fibrosis 3 Muscular dystrophy 4 Multiple sclerosis

2 The early symptom of cystic fibrosis is meconium ileus, which is impacted stool in the newborn. Thick mucous secretions, salty sweat, and difficulty gaining weight because of high caloric demands are characteristics of the condition. Cerebral palsy is a motor disorder caused by damage to the brain. Muscular dystrophy is a muscular disorder. Multiple sclerosis is a condition with progressive disintegration of the myelin sheath.

A client, experiencing an exacerbation of Crohn disease, is admitted to the hospital for intravenous steroid therapy. The nurse should not assign this client to a room with a roommate who has which illness? 1 Pancreatitis 2 Thrombophlebitis 3 Bacterial meningitis 4 Acute cholecystitis

3

A nurse in the postpartum unit must complete several interventions before a client's discharge from the hospital. The nurse plans to delegate some of the tasks to an unlicensed health care worker. Which activity must be performed by the nurse? 1 Taking the neonate's picture 2 Placing the infant car seat in the car 3 Comparing the identification bands of mother and infant 4 Preparing the discharge packet and distributing them to parents

3 It is the nurse's professional responsibility to compare the mother's and infant's identification bands one last time before discharge. This ensures that the correct infant is discharged with the mother. Taking the neonate's picture, placing the infant seat in the car, and preparing the discharge packets and distributing them to parents are all within the role of the nursing assistant and may be delegated safely.

At a routine monthly visit, while assessing a client who is in her 26th week of gestation, the nurse identifies the presence of striae gravidarum. The nurse describes this condition to the client as what? 1 Brownish blotches on the face 2 Purplish discoloration of the cervix 3 Reddish streaks on the abdomen and breasts 4 A black line running between the umbilicus and mons veneris

3 Reddish streaks on the abdomen and breasts are striae gravidarum; they occur as a result of stretching of the breast and abdominal skin. These are known as "stretch marks." Chloasma refers to the condition where brownish blotches develop on the face. Purplish discoloration of the cervix is Chadwick sign. A black line running between the umbilicus and mons veneris is the linea nigra.

A client is admitted for hypertension, and serum electrolyte studies have yielded abnormal results. The scheduled workup includes a scan for an aldosteronoma. What gland is affected in aldosteronoma? 1 Kidney cortex 2 Thyroid gland 3 Pituitary gland 4 Adrenal cortex

4 An aldosteronoma is an aldosterone-secreting adenoma of the adrenal cortex. An aldosteronoma is not a tumor of the kidney cortex. An aldosteronoma is not a tumor of the thyroid gland. An aldosteronoma is not a tumor of the pituitary gland.

A 23-year-old woman comes to the clinic for a Pap smear. After the examination, the client confides that her mother died of endometrial cancer 1 year ago and says that she is afraid that she will die of the same cancer. Which risk factor stated by the client after an education session on risk factors indicates that further teaching is needed? 1 Obesity 2 High-fat diet 3 Hypertension 4 Late-onset menarche

4 Early-onset, not late-onset, menarche is a risk factor for endometrial cancer. A high-fat diet, hypertension, and obesity are all risk factors for endometrial cancer.

The nurse is creating a dietary plan for a client with cholecystitis who has been placed on a modified diet. Which will be most appropriate to include in the client's dietary plan? 1 Offer soft-textured foods to reduce the digestive burden 2 Offer low-cholesterol foods to avoid further formation of gallstones 3 Increase protein intake to promote tissue healing and improve energy reserves 4 Decrease fat intake to avoid stimulation of the cholecystokinin mechanism for bile release

4 Fat intake stimulates cholecystokinin release that signals the gallbladder to contract, causing pain. Soft-textured foods are unnecessary. Eating low-cholesterol foods to avoid further formation of gallstones is not true for all clients with cholecystitis; low-cholesterol foods are necessary if the cholecystitis is precipitated by cholelithiasis and the stones are composed of cholesterol. An increase in protein intake is necessary to promote tissue healing and improve energy reserves after a cholecystectomy, but is not as important as fat intake for cholecystitis.

A pregnant client is scheduled for ultrasonography at the end of her first trimester. What should the nurse instruct her to do in preparation for the sonogram? 1 Empty her bladder. 2 Avoid eating for 8 hours. 3 Take a laxative the night before the test. 4 Increase fluid intake for 1 hour before the procedure.

4 In the first trimester when fluid fills the bladder, the uterus is pushed up toward the abdominal cavity for optimum ultrasound viewing. The bladder must be full, not empty, for better visualization of the uterus. The gastrointestinal tract is not involved in ultrasound preparation.

A pregnant client's blood test reveals an increased alpha-fetoprotein (AFP) level. Which condition does the nurse suspect that this result indicates? 1 Cystic fibrosis 2 Phenylketonuria 3 Down syndrome 4 Neural tube defect

4 Increased levels of alpha-fetoprotein in pregnant women have been found to reflect open neural tube defects such as spina bifida and anencephaly. Cystic fibrosis is a genetic defect that is not associated with the AFP level. A Guthrie test soon after ingestion of formula can determine whether an infant has phenylketonuria. Down syndrome is a chromosomal defect that is associated with a low AFP level.

What is the primary responsibility of a nurse teaching the pregnant adolescent? 1 Instructing her about the care of an infant 2 Informing her of the benefits of breast-feeding 3 Advising her to watch for danger signs of preeclampsia 4 Encouraging her to continue regularly scheduled prenatal care

4 It is not uncommon for adolescents to avoid prenatal care; many do not recognize the deleterious effect that lack of prenatal care can have on them and their infants. Instruction in the care of an infant can be done in the later part of pregnancy and reinforced during the postpartum period. Informing the client of the benefits of breast-feeding should come later in pregnancy but not before the client's feelings about breast-feeding have been ascertained. Advising the client to watch for danger signs of preeclampsia is necessary, but it is not the priority intervention at this time.

A client has edema in the lower extremities during the day, which disappears at night. With which medical problem does the nurse conclude this clinical finding is consistent? 1 Pulmonary edema 2 Myocardial infarction 3 Deep vein thrombosis 4 Right ventricular heart failure

4 Right ventricular heart failure causes increased pressure in the systemic venous system, which leads to a fluid shift into the interstitial spaces. Because of gravity, the lower extremities are first affected in an ambulatory client. Pulmonary edema results in severe respiratory distress and peripheral edema with pink frothy sputum. Myocardial infarction itself does not cause peripheral edema. The edema in deep vein thrombosis will be constant and not disappear at night; redness is usually present.

A resident in a long-term care facility is diagnosed with hepatitis B. Which intervention should the nurse implement with the staff caring for this client? A.Determine if all employees have had the hepatitis B vaccine series. B.Explain that this type of hepatitis can be transmitted when feeding the client. C.Assure the employees that they cannot contract hepatitis B when providing direct care. D.Tell the employees that wearing gloves and a gown are required when providing care.

A Hepatitis B vaccine should be administered to all health care providers (A). Hepatitis A (not hepatitis B) can be transmitted by fecal-oral contamination (B). There is a chance that staff could contract hepatitis B if exposed to the client's blood and/or body fluids; therefore, (C) is incorrect. There is no need to wear gloves and gowns except with blood or body fluid contact (D).

A client is admitted to the hospital with a diagnosis of severe acute diverticulitis. Which nursing intervention has the highest priority? A.Place the client on NPO status. B.Assess the client's temperature. C.Obtain a stool specimen. D.Administer IV fluids.

A A client with acute severe diverticulitis is at risk for peritonitis and intestinal obstruction and should be made NPO (A) to reduce risk of intestinal rupture. (B, C, and D) are important but are less of a priority than (A), which is implemented to prevent a severe complication.

In caring for a client with acute diverticulitis, which assessment data warrants immediate nursing intervention? A.The client has a rigid hard abdomen and elevated WBC. B.The client has left lower quadrant pain and an elevated temperature. C.The client is refusing to eat any of the meal and is complaining of nausea. D.The client has not had a bowel movement in 2 days and has a soft abdomen.

A A hard rigid abdomen and elevated WBC is indicative of peritonitis (A), which is a medical emergency and should be reported to the health care provider immediately. (B and C) are expected clinical manifestations of diverticulitis. (D) does not warrant immediate intervention.

An 81-year-old male client has emphysema. He lives at home with his cat and manages self-care with no difficulty. When making a home visit, the nurse notices that this client's tongue is somewhat cracked and his eyeballs appear sunken into his head. Which nursing intervention is indicated? A.Help the client determine ways to increase his fluid intake. B.Obtain an appointment for the client to have an eye examination. C.Instruct the client to use oxygen at night and increase the humidification. D.Schedule the client for tests to determine his sensitivity to cat hair.

A Clients with COPD should ingest 3 L of fluids daily but may experience a fluid deficit because of shortness of breath. The nurse should suggest creative methods to increase the intake of fluids (A), such as having fruit juices in disposable containers readily available. (B) is not indicated. Humidified oxygen will not effectively treat the client's fluid deficit, and there is no indication that the client needs supplemental oxygen at night (C). These symptoms are not indicative of (D) and may unnecessarily upset the client, who depends on his pet for socialization.

During the shift report, the charge nurse informs a nurse that she has been assigned to another unit for the day. The nurse begins to sigh deeply and tosses about her belongings as she prepares to leave, making it known that she is very unhappy about being floated to the other unit. What is the best immediate action for the charge nurse to take? A.Continue with the shift report and talk to the nurse about the incident at a later time. B.Ask the nurse to call the house supervisor to see if she must be reassigned. C.Stop the shift report and remind the nurse that all staff are floated equally. D.Inform the nurse that her behavior is disruptive to the rest of the staff.

A Continuing with the shift report (A) is the best immediate action because it allows the nurse who was floated some cooling off time. At a later time (after the nurse has cooled off) the charge nurse should discuss the conduct of the nurse in private. (B) encourages the nurse to shirk the float assignment. (C) is disruptive. Reprimanding the nurse in front of the staff would increase the nurse's hostility, so the nurse should be counseled in private (D).

After attending a class on reducing cancer risk factors, a client selects bran flakes with 2% milk and orange slices from a breakfast menu. In evaluating the client's learning, the nurse affirms that the client has made good choices and makes what additional recommendation? A.Switch to skim milk. B.Switch to orange juice. C.Add a source of protein. D.Add herbal tea.

A Dietary recommendations to reduce cancer risk include reduced consumption of fats, with increased consumption of fruits, vegetables, and fiber. (A) promotes reduced fat consumption. Orange slices provide more fiber than orange juice (B, C, and D) are not standard recommendations for reducing cancer risk.

23. A patient is having inspiratory stridor (crowing respiration) and the nurse suspects he is experiencing a laryngospasm. Which of the following would be most appropriate to implement for a patient experiencing a laryngospasm? A. Administer 100% oxygen. B. Position the patient in high Fowler's position. C. Insert a 16-gauge (large-bore) IV needle. D. Activate the emergency response team (code blue team) to the patient's room.

A. Administer 100% oxygen.A nurse should immediately administer 100% oxygen to the patient until the airway is fully reestablished, the larynx relaxes, and the spasms stop. Activating the emergency response team is not an immediate nursing action at this time because the nurse can administer the oxygen without the assistance of others. Positioning the patient in high Fowler's will not address the patient's need for immediate reoxygenation because of the patient's compromised respiratory state. Insertion of an IV device is not the first priority response but should be implemented after the nurse has assessed that the airway is stable.

4. When caring for a patient who is 3 hours postoperative laryngectomy, the nurse's highest priority assessment would be: A. Airway patency B. Patient comfort C. Incisional drainage D. Blood pressure and heart rate

A. Airway patency Remember ABCs with prioritization. Airway patency is always the highest priority and is essential for a patient undergoing surgery surrounding the upper respiratory system.

34. The patient has an order for each of the following inhalers. Which of the following should the nurse offer to the patient at the onset of an asthma attack? A. Albuterol (Proventil) B. Beclomethasone (Beclovent) C. Ipratropium bromide (Atrovent) D. Salmeterol (Serevent)

A. Albuterol (Proventil) Albuterol is a short-acting bronchodilator that should initially be given when the patient experiences an asthma attack.

21. A 45-year-old man with asthma is brought to the emergency department by automobile. He is short of breath and appears frightened. During the initial nursing assessment, which of the following clinical manifestations might be present as an early symptom during an exacerbation of asthma? A. Anxiety B. Cyanosis C. Hypercapnia D. Bradycardia

A. Anxiety An early symptom during an asthma attack is anxiety because he is acutely aware of the inability to get sufficient air to breathe. He will be hypoxic early on with decreased PaCO2 and increased pH as he is hyperventilating.

3. Following a patient's bone marrow aspiration, which of the following nursing interventions should a nurse anticipate? A. Application of firm pressure to the site B. Positioning the patient in a prone position C. Positioning the patient in a supine position D. Application of a warm, moist compress to the site

A. Application of firm pressure to the site After a bone marrow aspiration procedure, a nurse should apply pressure to the aspiration site until bleeding stops. Application of a warm, moist compress will not alter the potential for bleeding. Positioning the patient to assume a supine or prone position also will not address the need to control bleeding from the aspiration site.

5. When initially teaching a patient the supraglottic swallow following a radical neck dissection, with which of the following foods should the nurse begin? A. Cola B. Applesauce C. French fries D. White grape juice

A. ColaWhen learning the supraglottic swallow, it may be helpful to start with carbonated beverages because the effervescence provides clues about the liquid's position. Thin, watery fluids should be avoided because they are difficult to swallow and increase the risk of aspiration. Nonpourable pureed foods, such as applesauce, would decrease the risk of aspiration, but carbonated beverages are the better choice to start with.

3. When caring for a patient with metastatic cancer, the nurse notes a hemoglobin level of 8.7 g/dl and hematocrit of 26%. The nurse would place highest priority on initiating interventions that will reduce which of the following? A. Fatigue B. Thirst C. Headache D. Abdominal pain

A. Fatigue The patient with a low hemoglobin and hematocrit (normal values approximately 13.5% to 17% and 40% to 54%, respectively) is anemic and would be most likely to experience fatigue. This symptom develops because of the lowered oxygen-carrying capacity that leads to reduced tissue oxygenation to carry out cellular functions.

6. The nurse is caring for a patient admitted to the hospital with pneumonia. Upon assessment, the nurse notes a temperature of 101.4° F, a productive cough with yellow sputum and a respiratory rate of 20. Which of the following nursing diagnosis is most appropriate based upon this assessment? A. Hyperthermia related to infectious illness B. Ineffective thermoregulation related to chilling C. Ineffective breathing pattern related to pneumonia D. Ineffective airway clearance related to thick secretions

A. Hyperthermia related to infectious illness Because the patient has spiked a temperature and has a diagnosis of pneumonia, the logical nursing diagnosis is hyperthermia related to infectious illness. There is no evidence of a chill, and her breathing pattern is within normal limits at 20 breaths per minute. There is no evidence of ineffective airway clearance from the information given because the patient is expectorating sputum.

49. The nurse evaluates that a patient is experiencing the expected beneficial effects of ipratropium (Atrovent) after noting which of the following assessment findings? A. Increased peak flow readings B. Increased level of consciousness C. Decreased sputum production D. Increased respiratory rate

A. Increased peak flow readings. Ipratropium is a bronchodilator that should lead to increased PEFRs.

1. When assessing a patient's nutritional-metabolic pattern related to hematologic health, the nurse would: A. Inspect the skin for petechiae. B. Ask the patient about joint pain. C. Assess for vitamin C deficiency. D. Determine if the patient can perform ADLs.

A. Inspect the skin for petechiae. Any changes in the skin's texture or color should be explored when assessing the patient's nutritional-metabolic pattern related to hematologic health. The presences of petechiae or ecchymotic areas could be indicative of hematologic deficiencies related to poor nutritional intake or related causes.

35. The nurse who has administered a first dose of oral prednisone (Deltasone) to the patient with asthma writes on the care plan to begin monitoring which of the following patient parameters? A. Intake and output B. Bowel sounds C. Apical pulse D. Deep tendon reflexes

A. Intake and output Corticosteroids such as prednisone can lead to fluid retention. For this reason, it is important to monitor the patient's intake and output.

10. If a nurse is caring for an 80-year-old patient with a temperature of 100.4° F, crackles at the right lung base, pain with deep inspiration, and dyspnea, which of the following orders is the nurse's priority? A. Sputum specimen for culture and sensitivity B. Codeine 15 mg orally every 6 hours as needed C. Incentive spirometer every 2 hours while awake D. Amoxicillin (Amoxil) 500 mg orally 4 times a day

A. Sputum specimen for culture and sensitivity The patient presents with signs of a respiratory infection. To initiate the most effective therapy, the health care prescriber must know the pathogen causing the infection. Therefore, the sputum specimen is the nurse's priority. If the antibiotic is administered before the specimen is obtained, the results of the culture might not be as accurate and could impair the effectiveness of therapy. After the specimen is obtained, the nurse can administer codeine for coughing and begin the incentive spirometry to mobilize secretions and improve the patient's ability to expectorate the secretions.

9. Before beginning a transfusion of RBCs, which of the following actions by the nurse would be of highest priority to avoid an error during this procedure? A. Check the identifying information on the unit of blood against the patient's ID bracelet. B. Select new primary IV tubing primed with lactated Ringer's solution to use for the transfusion. C. Add the blood transfusion as a secondary line to the existing IV and used the IV controller to maintain correct flow. D. Remain with the patient for 60 minutes after beginning the transfusion to watch for signs of a transfusion reaction. The patient's identifying information (name, date of birth, medical record number) on the identification bracelet should exactly match the information on the blood bank tag that has been placed on the unit of blood. If any information does not match, the transfusions should not be hung because of possible error and risk to the patient.

A. The patient's identifying information (name, date of birth, medical record number) on the identification bracelet should exactly match the information on the blood bank tag that has been placed on the unit of blood. If any information does not match, the transfusions should not be hung because of possible error and risk to the patient.

19. The nurse is caring for a postoperative patient with sudden onset of respiratory distress. The physician orders a STAT ventilation-perfusion scan. Which of the following explanations should the nurse provide to the patient about the procedure? A. This test involves injection of a radioisotope to outline the blood vessels in the lungs, followed by inhalation of a radioisotope gas. B. This test will use special technology to examine cross sections of the chest with use of a contrast dye. C. This test will use magnetic fields to produce images of the lungs and chest. D. This test involves injecting contrast dye into a blood vessel to outline the blood vessels of the lungs.

A. This test involves injection of a radioisotope to outline the blood vessels in the lungs, followed by inhalation of a radioisotope gas.A ventilation-perfusion scan has two parts. In the perfusion portion, a radioisotope is injected into the blood and the pulmonary vasculature is outlined. In the ventilation part, the patient inhales a radioactive gas that outlines the alveoli.

27. A patient with acute exacerbation of COPD needs to receive precise amounts of oxygen. Which of the following types of equipment should the nurse prepare to use? A. Venturi mask B. Partial non-rebreather mask C. Oxygen tent D. Nasal cannula

A. Venturi mask The Venturi mask delivers precise concentrations of oxygen and should be selected whenever this is a priority concern. The other methods are less precise in terms of amount of oxygen delivered.

6. A nurse is working on a respiratory care unit where many of the patients are affected by asthma. Which of the following actions by the nurse would most likely increase respiratory difficulty for the patients? A. Wearing perfume to work B. Encouraging patients to ambulate daily C. Allowing the patients to eat green leafy vegetables D. Withholding antibiotic therapy until cultures are obtained

A. Wearing perfume to work People with asthma should avoid extrinsic allergens and irritants (e.g., dust, pollen, smoke, certain foods, colognes and perfumes, certain types of medications) because their airways become inflamed, producing shortness of breath, chest tightness, and wheezing. Many green leafy vegetables are rich in vitamins, minerals, and proteins, which incorporate healthy lifestyle patterns into the patients' daily living routines. Routine exercise is a part of a prudent lifestyle, and for patients with asthma the physical and psychosocial effects of ambulation can incorporate feelings of well-being, strength, and enhancement of physical endurance. Antibiotic therapy is always initiated after cultures are obtained so that the sensitivity to the organism can be readily identified.

24. The nurse is caring for a patient with an acute exacerbation of asthma. Following initial treatment, which of the following findings indicates to the nurse that the patient's respiratory status is improving? A. Wheezing becomes louder B. Vesicular breath sounds decrease C. Aerosol bronchodilators stimulate coughing D. The cough remains nonproductive

A. Wheezing becomes louder The primary problem during an exacerbation of asthma is narrowing of the airway and subsequent diminished air exchange. As the airways begin to dilate, wheezing gets louder because of better air exchange.

28. To find the infection site associated with acute lymphangitis, the nurse should look _____ to the inflammation. A. distal B. anterior C. proximal D. contralateral

A. distal The nurse should assess distal to swelling to locate the initial site of infection. Examining proximal, contralateral, or anterior to the inflammation does not describe swelling associated with infection.

12. A nurse is providing care to an adult female patient and observes that the Hb laboratory analysis result is 9 g/dl. Based on this finding, the nurse should expect to observe A. dyspnea. B. bradycardia. C. warm, dry skin. D. activity tolerance without complaint of fatigue.

A. dyspnea.Hb levels are used to determine the severity of anemia. Patients with moderate anemia (Hb 6 to 10 g/dL) may suffer from dyspnea, palpitations, diaphoresis with exertion, and chronic fatigue. Patients who are anemic usually have cool skin related to compensatory mechanism of mild vasoconstriction. Patients who are anemic experience tachycardia because of increased demands placed on the heart to meet overall metabolic requirements. Activity tolerance without complaint is not correct because patients with anemic conditions fatigue readily.

7. The nurse notes a physician's order written at 10:00 AM for 2 units of packed red blood cells to be administered to a patient who is anemic secondary to chronic blood loss. If the transfusion is picked up at 11:30, the nurse should plan to hang the unit no later than which of the following times? A. 11:45 AM B. 12:00 noon C. 12:30 PM D. 3:30 PM

B. 12:00 noon The nurse must hang the unit of packed red blood cells within 30 minutes of signing them out from the blood bank

35. A patient is admitted to the hospital with fever, chills, a productive cough with rusty sputum, and pleuritic chest pain. Pneumococcal pneumonia is suspected. An appropriate nursing diagnosis for the patient based on the patient's manifestations is A. hyperthermia related to acute infectious process. B. chronic pain related to ineffective pain management. C. risk for injury related to disorientation and confusion. D. ineffective airway clearance related to retained secretions.

A. hyperthermia related to acute infectious process. The patient with pneumococcal pneumonia is acutely ill with fever and the systemic manifestations of fever, such as chills, thirst, headache, and malaise. Interventions that monitor temperature and aid in lowering body temperature are appropriate. Ineffective airway clearance would be manifested by adventitious breath sounds and difficulty producing secretions. Disorientation and confusion are not noted in this patient and are not typical unless the patient is very hypoxemic. Pleuritic pain is an acute pain that is due to inflammation of the pleura.

16. To ensure the correct amount of oxygen delivery for a patient receiving 35% oxygen via a Venturi mask, it is most important that the nurse A. keep the air-entrainment ports clean and unobstructed. B. apply an adaptor to increase humidification of the oxygen. C. drain moisture condensation from the oxygen tubing every hour. D. keep the flow rate high enough to keep the bag from collapsing during inspiration.

A. keep the air-entrainment ports clean and unobstructed. Oxygen is delivered to a small jet in the center of a wide-based cone. Air is entrained (pulled through) openings in the cone as oxygen flows through the small jet. The degree of restriction or narrowness of the jet determines the amount of entrainment and the dilution of pure oxygen with room air and thus the concentration of oxygen. Although applying an adaptor can increase the humidification with the Venturi mask, it is not the best answer, because an open port is essential to proper functioning. Draining moisture condensation from the oxygen tubing is performed as often as needed, not on an hourly schedule. A plastic face mask with a reservoir bag needs to have sufficient flow rate to keep the bag inflated.

20. Using light pressure with the index and middle fingers, the nurse cannot palpate any of the patient's superficial lymph nodes. The nurse A. records this finding as normal. B. should reassess the lymph nodes using deeper pressure. C. asks the patient about any history of any radiation therapy. D. notifies the health care provider that x-rays of the nodes will be necessary.

A. records this finding as normal. Superficial lymph nodes are evaluated by light palpation, but they are not normally palpable. It may be normal to find small (<1.0 cm), mobile, firm, nontender nodes. Deep lymph nodes are detected radiographically.

17. While caring for a patient with respiratory disease, a nurse observes that the oxygen saturation drops from 94% to 85% when the patient ambulates. The nurse should determine that A. supplemental oxygen should be used when the patient exercises. B. ABG determinations should be done to verify the oxygen saturation reading. C. this finding is a normal response to activity and that the patient should continue to be monitored. D. the oximetry probe should be moved from the finger to the earlobe for an accurate oxygen saturation measurement during activity.

A. supplemental oxygen should be used when the patient exercises.An oxygen saturation lower than 90% indicates inadequate oxygenation. If the drop is related to activity of some type, supplemental oxygen is indicated.

3. The nurse notices clear nasal drainage in a patient newly admitted with facial trauma, including a nasal fracture. The nurse should: A. test the drainage for the presence of glucose. B. suction the nose to maintain airway clearance. C. document the findings and continue monitoring. D. apply a drip pad and reassure the patient this is normal.

A. test the drainage for the presence of glucose. Clear nasal drainage suggests leakage of cerebrospinal fluid (CSF). The drainage should be tested for the presence of glucose, which would indicate the presence of CSF.

43. Select all that apply. Which of the following is included in a comprehensive respiratory assessment? A. Pulse oximetry B. Chest auscultation C. Apical radial pulse D. Nail-bed assessment E. Evaluation of respiratory effort F. Rate and character of respirations

ABDEF The total assessment of the respiratory system includes pulse oximetry; auscultation; skin and nail-bed assessment for the detection of cyanosis; and rate, character, and degree of effort of respirations. The apical radial pulse is a cardiac assessment.

42. Select all that apply. Which of the following nursing actions can help clear tracheobronchial secretions in a patient with cystic fibrosis? A. Postural drainage B. Suppressing the cough C. Ensuring adequate hydration D. Administering mucolytic aerosols E. Encouraging the patient to lie flat F. Administering water-soluble vitamins

ACD Postural drainage, adequate hydration, and administration of mucolytic aerosols all encourage coughing and the clearing of secretions. A patient with cystic fibrosis will be more comfortable sitting upright.

9. If a health care provider is planning to transfuse a patient with a unit of packed red blood cells, which of the following solutions should the health care provider hang with the transfusion? A. 5% dextrose in water B. 0.9% sodium chloride C. 5% dextrose in 0.9% sodium chloride D. 5% dextrose in lactated Ringer's solution

B. 0.9% sodium chloride The only solution appropriate for administration with whole blood or blood products is 0.9% sodium chloride. The other options are not appropriate for use with blood products.

A client diagnosed with chronic kidney disease (CKD) 2 years ago is regularly treated at a community hemodialysis facility. Before his scheduled dialysis treatment, which electrolyte imbalance should the nurse anticipate? A.Hypophosphatemia B.Hypocalcemia C.Hyponatremia D.Hypokalemia

B Hypocalcemia (B) develops in CKD because of chronic hyperphosphatemia, not (A). Increased phosphate levels cause the peripheral deposition of calcium and resistance to vitamin D absorption needed for calcium absorption. Prior to dialysis, the nurse would expect to find the client hypernatremic and hyperkalemic, not with (C or D).

A hospitalized client is receiving nasogastric tube feedings via a small-bore tube and a continuous pump infusion. He begins to cough and produces a moderate amount of white sputum. Which action should the nurse take FIRST? A.Auscultate the client's breath sounds. B.Turn off the continuous feeding pump. C.Check placement of the nasogastric tube. D.Measure the amount of residual feeding.

B A productive cough may indicate that the feeding has been aspirated. The nurse should first stop the feeding (B) to prevent further aspiration. (A, C, and D) should all be performed before restarting the tube feeding if no evidence of aspiration is present and the tube is in place.

Which instruction should the nurse teach a female client about the prevention of toxic shock syndrome? A."Get immunization against human papillomavirus (HPV)." B."Change your tampon frequently." C."Empty your bladder after intercourse." D."Obtain a yearly flu vaccination."

B Certain strains of Staphylococcus aureus produce a toxin that can enter the bloodstream through the vaginal mucosa. Changing the tampon frequently (B) reduces the exposure to these toxins, which are the primary cause of toxic shock syndrome. (A) helps prevent cervical cancer, not toxic shock syndrome. (C) can lessen the incidence of urinary tract infection. (D) can help prevent some individuals from contracting the flu and pneumonia, but no relationship to toxic shock syndrome has been proven.

The nurse is reviewing routine medications taken by a client with chronic angle closure glaucoma. Which medication prescription should the nurse question? A.Antianginal with a therapeutic effect of vasodilation B.Anticholinergic with a side effect of pupillary dilation C.Antihistamine with a side effect of sedation D.Corticosteroid with a side effect of hyperglycemia

B Clients with angle-closure glaucoma should not take medications that dilate the pupil (B) because this can precipitate acute and severely increased intraocular pressure. (A, C, and D) do not cause increased intracranial pressure, which is the primary concern with angle-closure glaucoma.

A 58-year-old client who has no health problems asks the nurse about receiving the pneumococcal vaccine (Pneumovax). Which statement given by the nurse would offer the client accurate information about this vaccine? A.The vaccine is given annually before the flu season to those older than 50 years. B.The immunization is administered once to older adults or those at risk for illness. C.The vaccine is for all ages and is given primarily to those persons traveling overseas to areas of infection. D.The vaccine will prevent the occurrence of pneumococcal pneumonia for up to 5 years.

B It is usually recommended that persons older than 65 years and those with a history of chronic illness should receive the vaccine once in their lifetime (B). Some recommend receiving the vaccine at 50 years of age. The influenza vaccine is given once a year, not Pneumovax (A). Although the vaccine might be given to a person traveling overseas, that is not the main rationale for administering the vaccine (C). The vaccine is usually given once in a lifetime (D), but with immunosuppressed clients or clients with a history of pneumonia, revaccination is sometimes required.

Which abnormal laboratory finding indicates that a client with diabetes needs further evaluation for diabetic nephropathy? A.Hypokalemia B.Microalbuminuria C.Elevated serum lipid levels D.Ketonuria

B Microalbuminuria (B) is the earliest sign of diabetic nephropathy and indicates the need for follow-up evaluation. Hyperkalemia, not (A), is associated with end-stage renal disease caused by diabetic nephropathy. (C) may be elevated in end-stage renal disease. (D) may signal the onset of diabetic ketoacidosis (DKA).

The nurse assesses a postoperative client whose skin is cool, pale, and moist. The client is very restless and has scant urine output. Oxygen is being administered at 2 L/min, and a saline lock is in place. Which intervention should the nurse implement first? A.Measure the urine specific gravity. B.Obtain IV fluids for infusion per protocol. C.Prepare for insertion of a central venous catheter. D.Auscultate the client's breath sounds.

B The client is at risk for hypovolemic shock because of the postoperative status and is exhibiting early signs of shock. A priority intervention is the initiation of IV fluids (B) to restore tissue perfusion. (A, C, and D) are all important interventions, but are of less priority than (B).

A male client has just undergone a laryngectomy and has a cuffed tracheostomy tube in place. When initiating bolus tube feedings postoperatively, when should the nurse inflate the cuff? A.Immediately after feeding B.Just prior to tube feeding C.Continuous inflation is required D.Inflation is not required

B The cuff should be inflated before the feeding to block the trachea and prevent food from entering (B) if oral feedings are started while a cuffed tracheostomy tube is in place. It should remain inflated throughout the feeding to prevent aspiration of food into the respiratory system. (A and D) place the client at risk for aspiration. (C) places the client at risk for tracheal wall necrosis.

When assigning clients on a medical-surgical floor to an RN and a PN, it is best for the charge nurse to assign which client to the PN? A.A young adult with bacterial meningitis with recent seizures B.An older adult client with pneumonia and viral meningitis C.A female client in isolation with meningococcal meningitis D.A male client 1 day postoperative after drainage of a brain abscess

B The most stable client is (B). (A, C, and D) are all at high risk for increased intracranial pressure and require the expertise of the RN for assessment and management of care.

The nurse is planning care for a client with diabetes mellitus who has gangrene of the toes to the midfoot. Which goal should be included in this client's plan of care? A.Restore skin integrity. B.Prevent infection. C.Promote healing. D.Improve nutrition.

B The prevention of infection is a priority goal for this client (B). Gangrene is the result of necrosis (tissue death). If infection develops, there is insufficient circulation to fight the infection and the infection can result in osteomyelitis or sepsis. Because tissue death has already occurred, (A and C) are unattainable goals. (D) is important but of less priority than (B).

A client on telemetry has a pattern of uncontrolled atrial fibrillation with a rapid ventricular response. Based on this finding, the nurse anticipates assisting the physician with which treatment? A.Administer lidocaine,75 mg intravenous push. B.Perform synchronized cardioversion. C.Defibrillate the client as soon as possible. D.Administer atropine, 0.4 mg intravenous push.

B With uncontrolled atrial fibrillation, the treatment of choice is synchronized cardioversion (B) to convert the cardiac rhythm back to normal sinus rhythm. (A) is a medication used for ventricular dysrhythmias. (C) is not for a client with atrial fibrillation; it is reserved for clients with life-threatening dysrhythmias, such as ventricular fibrillation and unstable ventricular tachycardia. (D) is the drug of choice in symptomatic sinus bradycardia, not atrial fibrillation.

50. The nurse is teaching a patient how to self-administer ipratropium (Atrovent) via a metered dose inhaler. Which of the following instructions given by the nurse is most appropriate to help the patient learn proper inhalation technique? A. "Avoid shaking the inhaler before use." B. "Breathe out slowly before positioning the inhaler." C. "After taking a puff, hold the breath for 30 seconds before exhaling." D. "Using a spacer should be avoided for this type of medication."

B. "Breathe out slowly before positioning the inhaler." It is important to breathe out slowly before positioning the inhaler. This allows the patient to take a deeper breath while inhaling the medication thus enhancing the effectiveness of the dose.

47. A patient has been receiving oxygen per nasal cannula while hospitalized for COPD. The patient asks the nurse whether oxygen use will be needed at home. Which of the following would be the most appropriate response by the nurse? A. "Long-term home oxygen therapy should be used to prevent respiratory failure." B. "Oxygen will be needed when your oxygen saturation drops to 88% and you have symptoms of hypoxia. C. "Long-term home oxygen therapy should be used to prevent heart problems related to emphysema." D. "Oxygen will not be needed until or unless you are in the terminal stages of this disease."

B. "Oxygen will be needed when your oxygen saturation drops to 88% and you have symptoms of hypoxia.Long-term oxygen therapy in the home should be considered when the oxygen saturation is 88% or less and the patient has signs of tissue hypoxia, such as cor pulmonale, erythrocytosis, or impaired mental status.

24. A nurse is preparing to establish oxygen therapy for a patient with COPD, and the physician's prescription reads "oxygen per nasal cannula at 5 L per minute." Which of the following actions should the nurse take? A. Administer the oxygen as prescribed. B. Call the physician and question the correct flow rate of the oxygen. C. Establish the oxygen as prescribed and obtain an ABG. D. Change the delivery device from a nasal cannula to a simple oxygen mask.

B. Call the physician and question the correct flow rate of the oxygen. The nurse should call the physician immediately and question the flow rate for delivery of the oxygen before implementation. Oxygen is used cautiously in patients with COPD because of longstanding hypoxemia serving as the respiratory drive mechanism. If high levels of oxygen are administered, the respiratory drive can be obliterated. Changing the device to a simple oxygen mask may alter the oxygen concentration being delivered to the patient and will further enhance the obliteration of the patient's respiratory drive. Obtaining an ABG sample is not a priority at this time, and the action does not address the validity of the prescribed oxygen dosing for the patient.

20. If a patient states, "It's hard for me to breathe and I feel short-winded all the time," what is the most appropriate terminology to be applied in documenting this assessment by a nurse? A. Apnea B. Dyspnea C. Tachypnea D. Respiratory fatigue

B. Dyspnea Dyspnea is a subjective description reflective of the patient's statement indicating difficulty in breathing. Apnea refers to absence of breath or breathing. Tachypnea refers to an increased rate of breathing, usually greater than 20 breaths per minute. Respiratory fatigue is subjective and usually refers to the patient exhibiting signs and symptoms associated with a comprehensive respiratory assessment including laborious breathing, use of accessory muscles, and slowing of respirations.

9. Which of the following nursing interventions is of the highest priority in helping a patient expectorate thick secretions related to pneumonia? A. Humidify the oxygen as able B. Increase fluid intake to 3L/day if tolerated. C. Administer cough suppressant q4hr. D. Teach patient to splint the affected area.

B. Increase fluid intake to 3L/day if tolerated. Although several interventions may help the patient expectorate mucus, the highest priority should be on increasing fluid intake, which will liquefy the secretions so that the patient can expectorate them more easily. Humidifying the oxygen is also helpful, but is not the primary intervention. Teaching the patient to splint the affected area may also be helpful, but does not liquefy the secretions so that they can be removed.

8. The nurse receives a physician's order to transfuse fresh frozen plasma to a patient suffering from an acute blood loss. Which of the following procedures is most appropriate for infusing this blood product? A. Hand the fresh frozen plasma as a piggyback to a new bag of primary IV solution without KCl. B. Infuse the fresh frozen plasma as rapidly as the patient will tolerate. C. Hang the fresh frozen plasma as a piggyback to the primary IV solution. D. Infuse the fresh frozen plasma as a piggyback to a primary solution of normal saline.

B. Infuse the fresh frozen plasma as rapidly as the patient will tolerate. The fresh frozen plasma should be administered as rapidly as possible and should be used within 2 hours of thawing. Fresh frozen plasma is infused using any straight-line infusion set. Any existing IV should be interrupted while the fresh frozen plasma is infused, unless a second IV line has been started for the transfusion.

2. Which of the following is a factor significant in the development of anemia in men? A. Condom use B. Large hemorrhoids C. A diet high in cholesterol D. Smoking one pack of cigarettes daily

B. Large hemorrhoids Gastrointestinal (GI) tract bleeding is a common etiologic factor in men and may result from peptic ulcers, hiatal hernia, gastritis, cancer, hemorrhoids, diverticula, ulcerative colitis, or salicylate poisoning.

17. The nurse is caring for a 73-year-old patient who underwent a left total knee arthroplasty. On the third postoperative day, the patient complains of shortness of breath, slight chest pain, and that "something is wrong." Temperature is 98.4o F, blood pressure 130/88, respirations 36, and oxygen saturation 91% on room air. Which of the following should the nurse first suspect as the etiology of this episode? A. Septic embolus from the knee joint B. Pulmonary embolus from deep vein thrombosis C. New onset of angina pectoris D. Pleural effusion related to positioning in the operating room

B. Pulmonary embolus from deep vein thrombosis The patient presents the classic symptoms of pulmonary embolus: acute onset of symptoms, tachypnea, shortness of breath, and chest pain.

37. The nurse is scheduled to give a dose of salmeterol by metered dose inhaler (MDI). The nurse would administer the right drug by selecting the inhaler with which of the following trade names? A. Vanceril B. Serevent C. AeroBid D. Atrovent

B. Serevent The trade or brand name for salmeterol, an adrenergic bronchodilator, is Serevent.

26. The nurse is assigned to care for a patient in the emergency department admitted with an exacerbation of asthma. The patient has received a β-adrenergic bronchodilator and supplemental oxygen. If the patient's condition does not improve, the nurse should anticipate which of the following is likely to be the next step in treatment? A. Pulmonary function testing B. Systemic corticosteroids C. Biofeedback therapy D. Intravenous fluids

B. Systemic corticosteroids Systemic corticosteroids speed the resolution of asthma exacerbations and are indicated if the initial response to the β-adrenergic bronchodilator is insufficient.

48. Before discharge, the nurse discusses activity levels with a 61-year-old patient with COPD and pneumonia. Which of the following exercise goals is most appropriate once the patient is fully recovered from this episode of illness? A. Slightly increase activity over the current level. B. Walk for 20 minutes a day, keeping the pulse rate less than 130 beats per minute. C. Limit exercise to activities of daily living to conserve energy. D. Swim for 10 min/day, gradually increasing to 30 min/day.

B. Walk for 20 minutes a day, keeping the pulse rate less than 130 beats per minute. The patient will benefit from mild aerobic exercise that does not stress the cardiorespiratory system. The patient should be encouraged to walk for 20 min/day, keeping the pulse rate less than 75% to 80% of maximum heart rate (220 minus patient's age).

14. A 71-year-old patient is admitted with acute respiratory distress related to cor pulmonale. Which of the following nursing interventions is most appropriate during admission of this patient? A. Delay any physical assessment of the patient and review with the family the patient's history of respiratory problems. B. Perform a comprehensive health history with the patient to review prior respiratory problems. C. Perform a physical assessment of the respiratory system and ask specific questions related to this episode of respiratory distress. D. Complete a full physical examination to determine the effect of the respiratory distress on other body functions.

C. Perform a physical assessment of the respiratory system and ask specific questions related to this episode of respiratory distress.Because the patient is having respiratory difficulty, the nurse should ask specific questions about this episode and perform a physical assessment of this system. Further history taking and physical examination of other body systems can proceed once the patient's acute respiratory distress is being managed.

10. During discharge teaching for a 65-year-old patient with emphysema and pneumonia, which of the following vaccines should the nurse recommend the patient receive? A. S. aureus B. H. influenzae C. Pneumococcal D. Bacille Calmette-Guérin (BCG)

C. Pneumococcal The pneumococcal vaccine is important for patients with a history of heart or lung disease, recovering from a severe illness, age 65 or over, or living in a long-term care facility.

26. Which of the following conditions is manifested by unexplained shortness of breath and a high mortality rate? A. Bleeding ulcer B. Transient ischemia C. Pulmonary embolism D. MI

C. Pulmonary embolism A high mortality rate is associated with a pulmonary embolism. A pulmonary embolism is an obstruction of the pulmonary artery caused by an embolus. It presents with hypoxia, anxiety, restlessness, and shortness of breath. Bleeding ulcers, MI, and transient ischemia are not associated with such a high mortality rate.

12. If a nurse is assessing a patient whose recent blood gas determination indicated a pH of 7.32 and respirations are measured at 32 breaths/min, which of the following is the most appropriate nursing assessment? A. The rapid breathing is causing the low pH. B. The nurse should sedate the patient to slow down respirations. C. The rapid breathing is an attempt to compensate for the low pH. D. The nurse should give the patient a paper bag to breathe into to correct the low pH.

C. The rapid breathing is an attempt to compensate for the low pH. The respiratory system influences pH (acidity) through control of carbon dioxide exhalation. Thus, rapid breathing increases the pH. Breathing into a paper bag aids a patient who is hyperventilating; in respiratory alkalosis, it aids in lowering the pH. The use of sedation can cause respiratory depression and hypoventilation, resulting in an even lower pH.

33. A patient with an acute pharyngitis is seen at the clinic with fever and severe throat pain that affects swallowing. On inspection the throat is reddened and edematous with patchy yellow exudates. The nurse anticipates that collaborative management will include A. treatment with antibiotics. B. treatment with antifungal agents. C. a throat culture or rapid strep antigen test. D. treatment with medication only if the pharyngitis does not resolve in 3 to 4 days.

C. a throat culture or rapid strep antigen test. Although inadequately treated β-hemolytic streptococcal infections may lead to rheumatic heart disease or glomerulonephritis, antibiotic treatment is not recommended until strep infections are definitely diagnosed with culture or antigen tests. The manifestations of viral and bacterial infections are similar, and appearance is not diagnostic except when candidiasis is present.

32. An excess of carbon dioxide in the blood causes an increased respiratory rate and volume because CO2 A. displaces oxygen on hemoglobin, leading to a decreased PaO2. B. causes an increase in the amount of hydrogen ions available in the body. C. combines with water to form carbonic acid, lowering the pH of cerebrospinal fluid. D. directly stimulates chemoreceptors in the medulla to increase respiratory rate and volume.

C. combines with water to form carbonic acid, lowering the pH of cerebrospinal fluid. A combination of excess CO2 and H2O results in carbonic acid, which lowers the pH of the cerebrospinal fluid and stimulates an increase in the respiratory rate. Peripheral chemoreceptors in the carotid and aortic bodies also respond to increases in PaCO2 to stimulate the respiratory center. Excess CO2 does not increase the amount of hydrogen ions available in the body but does combine with the hydrogen of water to form an acid.

34. Following a supraglottic laryngectomy, the patient is taught how to use the supraglottic swallow to minimize the risk of aspiration. In teaching the patient about this technique, the nurse instructs the patient to A. perform Valsalva maneuver immediately after swallowing. B. breathe between each Valsalva maneuver and cough sequence. C. cough after swallowing to remove food from the top of the vocal cords. D. practice swallowing thin, watery fluids before attempting to swallow solid foods.

C. cough after swallowing to remove food from the top of the vocal cords. A supraglottic laryngectomy involves removal of the epiglottis and false vocal cords, and the removal of the epiglottis allows food to enter the trachea. Supraglottic swallowing requires performance of the Valsalva maneuver before placing food in the mouth and swallowing. The patient then coughs to remove food from the top of the vocal cords, swallows again, and then breathes after the food has been removed from the vocal cords.

11. If a patient has pernicious anemia, the nurse should provide information regarding A. frequent bouts of dyspnea. B. risks relative to dehydration. C. deficiency of intrinsic factor. D. lack of any effective treatment for this condition.

C. deficiency of intrinsic factor. Pernicious anemia is a type of anemia caused by failure of absorption of vitamin B12 (cobalamin). The most common cause is lack of intrinsic factor, a glucoprotein produced by the parietal cells of the gastric lining.

18. A nurse establishes the presence of a tension pneumothorax when assessment findings reveal a(n) A. absence of lung sounds on the affected side. B. inability to auscultate tracheal breath sounds. C. deviation of the trachea toward the side opposite the pneumothorax. D. shift of the point of maximal impulse (PMI) to the left, with bounding pulses.

C. deviation of the trachea toward the side opposite the pneumothorax. Tension pneumothorax is caused by rapid accumulation of air in the pleural space, causing severely high intrapleural pressure. This results in collapse of the lung, and the mediastinum shifts toward the unaffected side, which is subsequently compressed.

8. A person complains of fatigue and malaise and has a slight temperature elevation for 2 days before symptoms of influenza (fever, chest congestion, and productive cough) become noticeable. During the time immediately before the illness is diagnosed, the patient A. could avoid contracting the disease if treatment is begun with antibiotics. B. is unable to spread the disease because it is still in the incubation period. C. is in the prodromal stage and is highly contagious and able to spread the disease. D. has a nosocomial infection, which affects approximately two million individuals a year.

C. is in the prodromal stage and is highly contagious and able to spread the disease. The prodromal stage is a short period of time (hours to several days) immediately preceding the onset of an illness during which the patient is very contagious. Antibiotics are not effective against viral illnesses. The incubation period is the time from entry of the organism to the onset of symptoms and, in some viral illnesses, may be contagious. Nosocomial infections are those acquired in a hospital, and this scenario does not suggest the source of the infection.

7. During care of a patient with multiple myeloma, an important nursing intervention is A. limiting activity to prevent pathologic fractures. B. assessing for changes in size and characteristics of lymph nodes. C. maintaining a fluid intake of 3 to 4 L/day to dilute calcium load. D. administering narcotic analgesics continuously to control bone pain.

C. maintaining a fluid intake of 3 to 4 L/day to dilute calcium load. Adequate hydration must be maintained to minimize problems from hypercalcemia. The goal of a urinary output of 1.5 to 2 L/day requires an intake of 3 to 4 L/day.

5. Absorption of vitamin B12 may be decreased in older adults because of decreased A. intestinal motility. B. production of bile by the liver. C. production of intrinsic factor by the stomach. D. synthesis of cobalamin (vitamin B12) by intestinal bacteria.

C. production of intrinsic factor by the stomach. Older persons are at risk for deficiency of cobalamin (pernicious anemia) because of a naturally occurring reduction of the intrinsic factor by the stomach mucosa. Absorption of cobalamin relies on intrinsic factor. Both must be present for absorption. Megaloblastic anemia is related to folate dysfunction. Intestinal motility (peristalsis) is the motion that moves food down the GI tract. The rhythmic contractions of muscles cause wave-like motions. Lack of peristalsis is called "paralytic ileus." Bile is produced in the liver, is stored and concentrated in the gallbladder, and is released into the duodenum when fat is eaten. Bile emulsifies fats and prepares them for enzyme digestion in order for the nutrient to be absorbed into lymph and eventually into blood vessels to the liver. Vitamin K (the blood-clotting vitamin) is synthesized by intestinal bacteria.

9. In older adults, infection after exposure to respiratory illness is most likely to A. result in similar rates of infection as in the younger adult. B. be easily prevented with the use of antibiotics after being exposed. C. result in serious lower respiratory infection related to weakened respiratory muscles and fewer cilia. D. be less serious because the older adult has less contact with younger children who are most likely to carry serious infections.

C. result in serious lower respiratory infection related to weakened respiratory muscles and fewer cilia. Changes in the older adult respiratory system make older adults more susceptible to infections that can be very serious and life threatening. Use of antibiotics to "prevent" lung infections is not recommended and is ineffective for viral infections.

38. In teaching the patient with COPD about the need for physical exercise, the nurse informs the patient that A. all patients with COPD should be able to increase walking gradually up to 20 min/day. B. a bronchodilator inhaler should be used to relieve exercise-induced dyspnea immediately after exercise. C. shortness of breath is expected during exercise but should return to baseline within 5 minutes after the exercise. D. monitoring the heart rate before and after exercise is the best way to determine how much exercise can be tolerated.

C. shortness of breath is expected during exercise but should return to baseline within 5 minutes after the exercise.Shortness of breath usually increases during exercise, but the activity is not being overdone if breathing returns to baseline within 5 minutes after stopping. Bronchodilators can be administered 10 minutes before exercise but should not be administered for at least 5 minutes after activity to allow recovery. Patients are encouraged to walk 15 to 20 minutes a day with gradual increases, but actual patterns will depend on patient tolerance. Dyspnea most frequently limits exercise and is a better indication of exercise tolerance than is heart rate in the patient with COPD.

16. If a patient with blood type O Rh- is given AB Rh- blood, the nurse would expect A. the patient's Rh factor to react with the RBCs of the donor blood. B. no adverse reaction because the patient has no antibodies against the donor blood. C. the anti-A and anti-B antibodies in the patient's blood to hemolyze the donor blood. D. the anti-A and anti-B antibodies in the donor blood to hemolyze the patient's blood.

C. the anti-A and anti-B antibodies in the patient's blood to hemolyze the donor blood. A patient with O Rh+ blood has no A or B antigens on the red cell but does have anti-A and anti-B antibodies in the blood and has an Rh antigen. AB Rh- blood has both A and B antigens on the red cell but no Rh antigen and no anti-A or anti-B antibodies. If the AB Rh- blood is given to the patient with O Rh+ blood, the antibodies in the patient's blood will react with the antigens in the donor blood, causing hemolysis of the donor cells. There will be no Rh reaction because the donor blood has no Rh antigen.

14. Upon entering the room of a patient who has just returned from surgery for total laryngectomy and radical neck dissection, a nurse should recognize a need for intervention when finding A. a gastrostomy tube that is clamped. B. the patient coughing blood-tinged secretions from the tracheostomy. C. the patient positioned in a lateral position with the head of the bed flat. D. 200 ml of serosanguineous drainage in the patient's portable drainage device.

C. the patient positioned in a lateral position with the head of the bed flat. After total laryngectomy and radical neck dissection, a patient should be placed in a semi-Fowler's position to decrease edema and limit tension on the suture line.

A practical nurse (PN) tells the charge nurse in a long-term facility that she does not want to be assigned to one particular resident. She reports that the male client keeps insisting that she is his daughter and begs her to stay in his room. What is the best managerial decision? A.Notify the family that the resident will have to be discharged if his behavior does not improve. B.Notify administration of the PN's insubordination and need for counseling about her statements. C.Ask the PN what she has done to encourage the resident to believe that she is his daughter. D.Reassign the PN until the resident can be assessed more completely for reality orientation.

D Temporary reassignment (D) is the best option until the resident can be examined and his medications reviewed. He may have worsening cerebral dysfunction from an infection or electrolyte imbalance. (A) is not the best option because the family cannot control the resident's actions. The administration may need to know about the situation, but not as a case of insubordination (B). Implying that the PN is somehow creating the situation is inappropriate until a further evaluation has been conducted (C).

The nurse on a medical surgical unit is receiving a client from the postanesthesia care unit (PACU) with a Penrose drain. Before choosing a room for this client, which information is most important for the nurse to obtain? A.If suctioning will be needed for drainage of the wound B.If the family would prefer a private or semiprivate room C.If the client also has a Hemovac in place D.If the client's wound is infected

D The fact that the client has a Penrose drain should alert the nurse to the possibility that the surgical wound is infected (D). Penrose drains provide a sinus tract or opening and are often used to provide drainage of an abscess. To avoid contamination of another postoperative client, it is most important to place any client with an infected wound in a private room. A Penrose drain does not require (A). Although (B) is helpful information, it does not have the priority of (D). A Hemovac (C) is used to drain fluid from a dead space and is not a determinant for the room assignment.

The nurse notes that the client's drainage has decreased from 50 to 5 mL/hr 12 hours after chest tube insertion for hemothorax. What is the best initial action for the nurse to take? A.Document this expected decrease in drainage. B.Clamp the chest tube while assessing for air leaks. C.Milk the tube to remove any excessive blood clot buildup. D.Assess for kinks or dependent loops in the tubing.

D The least invasive nursing action should be performed first to determine why the drainage has diminished (D). (A) is completed after assessing for any problems causing the decrease in drainage. (B) is no longer considered standard protocol because the increase in pressure may be harmful to the client. (C) is an appropriate nursing action after the tube has been assessed for kinks or dependent loops.

An older client is admitted with a diagnosis of bacterial pneumonia. Which symptom should the nurse report to the health care provider after assessing the client? A.Leukocytosis and febrile B.Polycythemia and crackles C.Pharyngitis and sputum production D.Confusion and tachycardia

D The onset of pneumonia in the older client may be signaled by general deterioration, confusion, increased heart rate, and/or increased respiratory rate (D). (A, B, and C) are often absent in the older client with bacterial pneumonia.

The nurse is giving preoperative instructions to a 14-year-old client scheduled for surgery to correct a spinal curvature. Which statement by the client best demonstrates that learning has taken place? A."I will read all the teaching booklets you gave me before surgery." B."I have had surgery before, so I know what to expect afterward." C."All the things people have told me will help me take care of my back." D."Let me show you the method of turning I will use after surgery."

D The outcome of learning is best demonstrated when the client not only verbalizes an understanding but can also provide a return demonstration (D). A 14-year-old client may or may not follow through with (A), and there is no measurement of learning. (B) may help the client understand the surgical process, but the type of surgery may have been very different, with differing postoperative care. In (C), the client may be saying what the nurse wants to hear without expressing any real understanding of what to do after surgery.

Which nursing action is necessary for the client with a flail chest? A.Withhold prescribed analgesic medications. B.Percuss the fractured rib area with light taps. C.Avoid implementing pulmonary suctioning. D.Encourage coughing and deep breathing.

D Treatment of flail chest is focused on preventing atelectasis and related complications of compromised ventilation by encouraging coughing and deep breathing (D). This condition is typically diagnosed in clients with three or more rib fractures, resulting in paradoxic movement of a segment of the chest wall. (C) should not be avoided because suctioning is necessary to maintain pulmonary toilet in clients who require mechanical ventilation. (A) should not be withheld. (B) should not be applied because the fractures are clearly visible on the chest radiograph.

11. The nurse evaluates that discharge teaching for a patient hospitalized with pneumonia has been most effective when the patient states which of the following measures to prevent a relapse? A. "I will increase my food intake to 2400 calories a day to keep my immune system well." B. "I must use home oxygen therapy for 3 months and then will have a chest x-ray to reevaluate." C. "I will seek immediate medical treatment for any upper respiratory infections." D. "I should continue to do deep-breathing and coughing exercises for at least 6 weeks."

D. "I should continue to do deep-breathing and coughing exercises for at least 6 weeks." It is important for the patient to continue with coughing and deep breathing exercises for 6 to 8 weeks until all of the infection has cleared from the lungs. A patient should seek medical treatment for upper respiratory infections that persist for more than 7 days. Increased fluid intake, not caloric intake, is required to liquefy secretions. Home O2 is not a requirement unless the patient's oxygenation saturation is below normal.

17. The nurse evaluates that teaching for the patient with iron deficiency anemia has been effective when the patient states A. "I will need to take the iron supplements the rest of my life." B. "I will increase my dietary intake of milk and milk products." C. "I should increase my activity to increase my aerobic capacity." D. "I should take the iron for several months after my blood is normal."

D. "I should take the iron for several months after my blood is normal." To replace the body's iron stores, iron supplements should be continued for 2 to 3 months after the Hb level returns to normal, but if the cause of the iron deficiency is corrected, the supplements do not need to be taken for a lifetime. Milk and milk products are poor sources of dietary iron. Activity should be gradually increased as Hb levels return to normal because aerobic capacity can be increased when adequate Hb is available.

44. When caring for a patient with COPD, the nurse identifies a nursing diagnosis of imbalanced nutrition less than body requirements after noting a weight loss of 30 lb. Which of the following would be an appropriate intervention to add to the plan of care for this patient? A. Teach the patient to use frozen meals at home that can be microwaved. B. Provide a high-calorie, high-carbohydrate, nonirritating, frequent feeding diet. C. Order fruits and fruit juices to be offered between meals. D. Order a high-calorie, high-protein diet with six small meals a day.

D. Order a high-calorie, high-protein diet with six small meals a day.Because the patient with COPD needs to use greater energy to breathe, there is often decreased oral intake because of dyspnea. A full stomach also impairs the ability of the diaphragm to descend during inspiration, interfering with the work of breathing. Finally, the metabolism of a high carbohydrate diet yields large amounts of CO2, which may lead to acidosis in patients with pulmonary disease. For these reasons, the patient with emphysema should take in a high-calorie, high-protein diet, eating six small meals per day.

13. Which of the following nursing interventions is most appropriate to enhance oxygenation in a patient with unilateral malignant lung disease? A. Positioning patient on right side. B. Maintaining adequate fluid intake C. Performing postural drainage every 4 hours D. Positioning patient with "good lung down"

D. Positioning patient with "good lung down" Therapeutic positioning identifies the best position for the patient assuring stable oxygenation status. Research indicates that positioning the patient with the unaffected lung (good lung) dependent best promotes oxygenation in patients with unilateral lung disease. For bilateral lung disease, the right lung down has best ventilation and perfusion. Increasing fluid intake and performing postural drainage will facilitate airway clearance, but positioning is most appropriate to enhance oxygenation.

33. The nurse determines that the patient is not experiencing adverse effects of albuterol (Proventil) after noting which of the following patient vital signs? A. Oxygen saturation 96% B. Respiratory rate of 18 C. Temperature of 98.4° F D. Pulse rate of 76

D. Pulse rate of 76 Albuterol is a β2-agonist that can sometimes cause adverse cardiovascular effects. These would include tachycardia and angina. A pulse rate of 76 indicates that the patient did not experience tachycardia as an adverse effect.

36. The nurse is assisting a patient to learn self-administration of beclomethasone two puffs inhalation q6hr. The nurse explains that the best way to prevent oral infection while taking this medication is to do which of the following as part of the self-administration techniques? A. Chew a hard candy before the first puff of medication. B. Ask for a breath mint following the second puff of medication. C. Rinse the mouth with water before each puff of medication. D. Rinse the mouth with water following the second puff of medication.

D. Rinse the mouth with water following the second puff of medication. The patient should rinse the mouth with water following the second puff of medication to reduce the risk of fungal overgrowth and oral infection.

18. In the case of pulmonary embolus from deep vein thrombosis, which of the following actions should the nurse take first? A. Notify the physician. B. Administer a nitroglycerin tablet sublingually. C. Conduct a thorough assessment of the chest pain. D. Sit the patient up in bed as tolerated and apply oxygen.

D. Sit the patient up in bed as tolerated and apply oxygen.The patient's clinical picture is consistent with pulmonary embolus, and the first action the nurse takes should be to assist the patient. For this reason, the nurse should sit the patient up as tolerated and apply oxygen before notifying the physician.

12. After admitting a patient to the medical unit with a diagnosis of pneumonia, the nurse will verify that which of the following physician orders have been completed before administering a dose of cefotetan (Cefotan) to the patient? A. Serum laboratory studies ordered for AM B. Pulmonary function evaluation C. Orthostatic blood pressures D. Sputum culture and sensitivity

D. Sputum culture and sensitivityThe nurse should ensure that the sputum for culture and sensitivity was sent to the laboratory before administering the cefotetan. It is important that the organisms are correctly identified (by the culture) before their numbers are affected by the antibiotic; the test will also determine whether the proper antibiotic has been ordered (sensitivity testing). Although antibiotic administration should not be unduly delayed while waiting for the patient to expectorate sputum, all of the other options will not be affected by the administration of antibiotics.

The nurse assesses a patient with shortness of breath for evidence of long-standing hypoxemia by inspecting: A. Chest excursion B. Spinal curvatures C. The respiratory pattern D. The fingernail and its base

D. The fingernail and its base Clubbing, a sign of long-standing hypoxemia, is evidenced by an increase in the angle between the base of the nail and the fingernail to 180 degrees or more, usually accompanied by an increase in the depth, bulk, and sponginess of the end of the finger.

25. The nurse identifies the nursing diagnosis of activity intolerance for a patient with asthma. The nurse assesses for which of the following etiologic factor for this nursing diagnosis in patients with asthma? A. Anxiety and restlessness B. Effects of medications C. Fear of suffocation D. Work of breathing

D. Work of breathingWhen the patient does not have sufficient gas exchange to engage in activity, the etiologic factor is often the work of breathing. When patients with asthma do not have effective respirations, they use all available energy to breathe and have little left over for purposeful activity.

21. To prevent atelectasis in an 82-year-old patient with a hip fracture, a nurse should A. supply oxygen. B. suction the upper airway. C. ambulate the patient frequently. D. assist the patient with aggressive coughing and deep breathing.

D. assist the patient with aggressive coughing and deep breathing. Decreased mobility after surgery in older adults creates the possibility of fluid buildup and retention in lung tissue. One of the primary goals of nursing intervention is to prevent atelectasis in a high-risk patient. Aggressive coughing and deep breathing can prevent atelectasis in the postoperative patient.

Misoprostol

PGE1 derivative: orally active prostaglandin used to prevent peptic ulcers in patients taking NSAIDs for arthritis. Tox: diarrhea

pursed-lip breathing

a technique of exhaling against pursed lips to prolong exhalation, preventing bronchiolar collapse and air trapping; done to increase expiratory airway pressure, improve oxygenation of the blood, and help prevent early airway closure.

pulmonary embolism

a thromboembolic occlusion of the pulmonary vasculature resulting from thrombi in the venous circulation or right side of the heart and from other sources, such as amniotic fluid, air, fat, bone marrow, and foreign IV material that travel as emboli until lodging in the pulmonary vessels.

An emaciated homeless client presents to the emergency department complaining of a productive cough, with blood-tinged sputum and night sweats. Which action is most important for the emergency department triage nurse to implement for this client? A.Initiate airborne infection precautions. B.Place a surgical mask on the client. C.Don an isolation gown and latex gloves. D.Start protective (reverse) isolation precautions.

This client is exhibiting classic symptoms of tuberculosis (TB), and the client is from a high-risk population for TB. Therefore, airborne infection precautions (A), which are indicated for TB, should be used with this client. (B) is used with DROPLET precautions. There is no evidence that (C or D) would be warranted at this time.

obstructive sleep apnea

a condition characterized by partial or complete upper airway obstruction during sleep, causing apnea and hypopnea.

chylothorax

a condition marked by lymphatic fluid in the pleural space caused by a leak in the thoracic duct.

pneumoconiosis

a general term for lung diseases caused by inhalation and retention of dust particles.

lung abscess

a pus-containing lesion of the lung parenchyma that results in a cavity formed by necrosis of lung tissue.

Legg-Calvé-Perthes disease

a self-limiting disorder in which there is aseptic necrosis of the femoral head. The aims of treatment are to eliminate hip irritability; restore and maintain adequate range of hip motion; prevent capital femoral epiphyseal collapse, extrusion, or subluxation; and ensure a well-rounded femoral head at the time of healing.

alpha 1-antitrypsin

a serum protein produced by the liver normally found in the lungs that inhibits proteolytic enzymes of white cells from lysing lung tissue; genetic deficiency of this protein can cause emphysema.

status asthmaticus

a severe, life-threatening asthma attack that is refractory to usual treatment and places the patient at risk for developing respiratory failure.

trigger

a substance, object, or agent that initiates or stimulates an action; in asthma, any stimuli that initiates the IgE

tracheotomy

a surgical incision into the trachea for the purpose of establishing an airway; performed below a blockage by a foreign body, tumor, or edema of the glottis.

thoracentesis

a surgical procedure done to remove fluid from the pleural space.

Secondary Preventive Care

-Goal is to detect and treat illnesses in early stages with screenings -Blood Pressure/ HIV/Glaucoma/Cholesterol, Pap Smears, Mammograms

Prinzmetals variant angina

-Unpredictable -Caused by coronary artery vasospasm

Which type of cranial surgery involves opening the cranium with a drill? 1 Burr hole 2 Craniotomy 3 Craniectomy 4 Cranioplasty

1 A burr hole involves the opening of the cranium using a drill. A craniotomy is a cranial surgery that involves opening the cranium with the removal of the bone flap and opening the dura to remove the lesion. A craniectomy is an excision into the cranium to cut away a bone flap. A cranioplasty is the repair of a cranial defect caused by trauma.

A pregnant client is admitted with abdominal pain and heavy vaginal bleeding. What is the priority nursing action? 1 Administering oxygen 2 Elevating the head of the bed 3 Drawing blood for a hematocrit level 4 Giving an intramuscular analgesic

1 Abdominal pain and heavy vaginal bleeding indicate significant blood loss. To compensate for decreased cardiac output, oxygen is given to maintain the well-being of both mother and fetus. Elevating the head of the bed will decrease blood flow to vital centers in the brain. Drawing blood for a hematocrit level is not the priority. Giving an intramuscular analgesic may mask abdominal pain and sedate an already compromised fetus; also, it requires a primary healthcare provider's prescription.

Two days after having a cesarean birth, a client tells the nurse that she has pain in her right leg. After an assessment the nurse suspects that the client has a thrombus. What is the nurse's primary response at this time? 1 Maintaining bed rest 2 Applying warm soaks 3 Performing leg exercises 4 Massaging the affected area

1 Although thrombophlebitis is suspected, before a definitive diagnosis can be made the client should be confined to bed so that further complications may be avoided. Applying warm soaks may cause vasodilation, which could allow a thrombus to dislodge and circulate freely. If a thrombus is present, massage may dislodge it and lead to a pulmonary embolism.

A client with ascites has a paracentesis, and 1500 mL of fluid is removed. For which immediate response is it most important for the nurse to monitor? 1 Rapid, thready pulse 2 Decreased peristalsis 3 Respiratory congestion 4 Increase in temperature

1 Fluid shifts from the intravascular compartment into the abdominal cavity, causing hypovolemia. A rapid, thready pulse [1] [2], which is indicative of shock, is a compensatory response to this shift. Decreased peristalsis is not likely to occur in the immediate period. After a paracentesis, intravascular fluid shifts into the abdominal cavity, not into the lungs. Increase in temperature is not the priority; body temperature usually is not affected immediately; an infection will take several days.

A client in the birthing suite has spontaneous rupture of the membranes, after which a prolapsed cord is identified. The nurse calls for help and with a sterile gloved hand moves the fetal head off the cord. What should the nurse anticipate? 1 Cesarean birth 2 Prolonged labor 3 Rapidly induced labor 4 Vacuum extraction vaginal birth

1 Immediate birth is necessary to prevent fetal hypoxia and death. Allowing a prolonged labor, inducing labor, or using vacuum extraction in a vaginal birth will increase pressure on the cord, resulting in fetal hypoxia.

To help prevent a cycle of recurring urinary tract infections in a female client, which instruction should the nurse share? 1 "Urinate as soon as possible after intercourse." 2 "Increase your daily intake of citrus juice." 3 "Douche regularly with alkaline agents." 4 "Take bubble baths regularly."

1 Intercourse may cause urethral inflammation, increasing the risk of infection; voiding clears the urinary meatus and urethra of microorganisms. Most fruit juices, with the exception of cranberry juice, cause alkaline urine, which promotes bacterial growth. Douching is no longer recommended because it alters the vaginal flora. Bubble baths can promote urinary tract infections.

What is the function of limbic system? 1 Influence emotional behavior 2 Regulate autonomic functions 3 Facilitate automatic movements 4 Relay sensory and motor inputs for cerebrum

1 Located lateral to the hypothalamus, the limbic system influences emotional behavior and basic drives such as feeding and sexual behaviors. The regulation of endocrine and autonomic functions is the function of the hypothalamus. The control and facilitation of learned and automatic movements is the function of the basal ganglia. The thalamus relays sensory and motor input to and from the cerebrum.

A client at 10 weeks' gestation calls the clinic and tells a nurse that she has morning sickness and cannot control it. What should the nurse suggest to promote relief? 1 "Eat dry crackers before you get out of bed." 2 "Increase your fat intake before bedtime." 3 "Drink high-carbohydrate fluids with meals." 4 "Eat two small meals a day and a snack at noon."

1 Nausea and vomiting in the morning occur in almost 50% of all pregnancies. Eating dry crackers before getting out of bed in the morning is a simple remedy that may provide relief. Increasing fat intake does not relieve the nausea. Drinking high-carbohydrate fluids with meals is not helpful; separating fluids from solids at mealtime is more advisable. Eating two small meals a day and a snack at noon does not meet the nutritional needs of a pregnant woman, nor will it relieve nausea. Some women find that eating five or six small meals daily instead of three large ones is helpful.

A client presents to the emergency room with coughing and sudden wheezing. The nurse notes the client is progressing quickly into respiratory distress. The nurse identifies that the client is experiencing what problem? 1 An acute asthma attack 2 Acute bronchitis 3 Left-sided heart failure 4 Cor pulmonale

1 Symptoms for an acute asthma attack often are wheezing, coughing, dyspnea, and chest tightness. Cough, fever, and fatigue are often symptoms exhibited with acute bronchitis. Fatigue, breathlessness, weakness, shortness of breath, and fluid accumulation in the lungs are often signs of left-sided heart failure. Tiring easily, shortness of breath with exertion, lower leg edema, chest pain, and heart palpitations often are exhibited with cor pulmonale.

A client is to undergo amniocentesis at 38 weeks' gestation to determine fetal lung maturity. What lecithin/sphingomyelin ratio (L/S ratio) is adequate for the nurse to conclude that the fetus's lungs are mature enough to sustain extrauterine life? 1 2:1 2 1:1 3 1:4 4 3:4

1 The lecithin concentration increases abruptly at 35 weeks, reaching a level that is twice the amount of sphingomyelin, which decreases concurrently. At 30 to 32 weeks' gestation, the amounts of lecithin and sphingomyelin are equal, indicating lung immaturity. A ratio of 1:4 does not reflect fetal lung maturity; nor does a ratio of 3:4.

Which phase of the woman's sexual response is characterized by elevation of the uterus? 1 Plateau phase 2 Orgasmic phase 3 Excitation phase 4 Resolution phase

1 The plateau phase occurs after the excitation phase, and excitation is maintained through the plateau phase, wherein the vagina expands and the uterus is elevated. Therefore elevation of the uterus is a characteristic of the plateau phase of a woman's sexual response. The orgasmic phase is characterized by uterine and vaginal contractions. In the excitation phase, the clitoris is congested and vaginal lubrication increases. The resolution phase is characterized by returning to the preexisting state.

nurse is assessing two clients. One client has ulcerative colitis, and the other client has Crohn disease. Which is more likely to be identified in the client with ulcerative colitis than in the client with Crohn disease? 1 Inclusion of transmural involvement of the small bowel wall 2 Higher occurrence of fistulas and abscesses from changes in the bowel wall 3 Pathology beginning proximally with intermittent plaques found along the colon 4 Involvement starting distally with rectal bleeding that spreads continuously up the colon

1 Ulcerative colitis involvement starts distally with rectal bleeding that spreads continuously up the colon to the cecum. In ulcerative colitis, pathology usually is in the descending colon; in Crohn disease, it is primarily in the terminal ileum, cecum, and ascending colon. Ulcerative colitis, as the name implies, affects the colon, not the small intestine. Intermittent areas of pathology occur in Crohn. In ulcerative colitis, the pathology is in the inner layer and does not extend throughout the entire bowel wall; therefore, abscesses and fistulas are rare. Abscesses and fistulas occur more frequently in Crohn disease.

What are the most common hormones produced in excess with hyperpituitarism? Select all that apply. 1 Prolactin 2 Growth hormone 3 Luteinizing hormone 4 Antidiuretic hormone 5 Melanocyte-stimulating hormone

1 2 The most common hormones produced in excess with hyperpituitarism are prolactin and growth hormone. Excessive stimulation of luteinizing hormone and antidiuretic hormone is also associated with hyperpituitarism, but less commonly than prolactin and growth hormone. Secretion of melanocyte-stimulating hormone stimulates adrenocorticotropic hormone, which indirectly stimulates the pituitary gland, thus leading to hyperpituitarism.

Which hormones are involved in building and maintaining healthy bone tissue? Select all that apply. 1 Insulin 2 Thyroxine 3 Glucocorticoids 4 Growth hormone 5 Parathyroid hormone

1, 4 Insulin works together with growth hormone to increase bone length, which helps to build and maintain healthy bone tissue. Thyroxine increases the rate of protein synthesis in all types of tissues. Glucocorticoids regulate protein metabolism to reduce or intensify the organic matrix of bone. Parathyroid hormone secretion stimulates bones to promote osteoclastic activity and release calcium into the blood when serum calcium levels are lowered.

Which gerontologic assessment findings of the auditory system are related to the inner ear? Select all that apply. 1 Hair cell degeneration 2 Reduced blood supply to the cochlea 3 Atrophic changes of the tympanic membrane 4 Decline in the ability to filter out unwanted sounds 5 Less effective vestibular apparatus in the semicircular canals

1,2,5 Hair cell degeneration, reduced blood supply to the cochlea, and less effective vestibular apparatus in the semicircular canals are assessment findings related to the inner ear. Atrophic changes of the tympanic membrane is an assessment finding associated with the middle ear. A decline in an ability to filter out unwanted sounds is an assessment finding related to the brain.

The nurse assists a client to the bathroom to void several times during the first stage of labor. Why is this is an important component of nursing? 1 A full bladder is often injured during labor. 2 A full bladder may inhibit the progress of labor. 3 A full bladder jeopardizes the status of the fetus. 4 A full bladder predisposes the client to urinary infection

2 A full bladder encroaches on the uterine space and impedes the descent of the fetal head. The bladder may become atonic, but is not physically damaged during the course of labor. A full bladder may lead to prolonged labor, but generally does not jeopardize fetal status as long as adequate placental perfusion continues. A full bladder during labor does not predispose the client to infection.

A client is rooming in with her newborn. The nurse observes the infant lying quietly in the bassinet with the eyes open wide. What action should the nurse take in response to the infant's behavior? 1 Brightening the lights in the room 2 Encouraging the mother to talk to her baby 3 Wrapping and then turning the infant to the side 4 Beginning physical and behavioral assessments

2 A quiet, alert state is an optimal time for infant stimulation. Bright lights are disturbing to newborns and may impede mother-infant interaction. Wrapping and then turning the infant to the side is done for the sleeping infant. Physical and behavioral assessments are not the priorities; they may be delayed.

The nurse is educating new parents about circumcision. Which structure of the penis would this nurse tell the parents is removed during circumcision? 1 Glans 2 Prepuce 3 Epididymis 4 Vas deferens

2 Circumcision is a procedure that involves removal of the prepuce, a skin fold over the glans. The glans is the tip of the penis. The epididymis is the internal structure that promotes transportation of the sperm. The vas deferens carries the sperm from the epididymis to the ejaculatory duct.

Which hormone increases the rate of protein synthesis in a client? 1 Estrogen 2 Thyroxine 3 Parathormone 4 Vitamin D

2 Thyroxine increases the rate of protein synthesis in all the body tissues. Estrogen stimulates bone-building, which is known as osteoblastic activity. Parathormone promotes osteoclastic activity in a state of hypocalcemia. Vitamin D and its metabolites are produced in the body and transported in the blood to promote the absorption of calcium and phosphorus from the small intestine.

A nurse is obtaining a health history from a client with the diagnosis of peptic ulcer disease. Which client statement provides evidence to support the identification of a possible contributory factor? 1 "My blood type is A positive." 2 "I smoke one pack of cigarettes a day." 3 "I have been overweight most of my life." 4 "My blood pressure has been high lately."

2 "I smoke one pack of cigarettes a day." Smoking cigarettes increases the acidity of gastrointestinal secretions, which damages the mucosal lining. While blood type O is more frequently associated with duodenal ulcer, type A has no significance. Being overweight is unrelated to peptic ulcer disease. High blood pressure is not directly related to peptic ulcer disease.

A nurse receives a call from the emergency department about a client with tuberculosis (TB) who will be admitted to the medical unit. Which precaution should the nurse take? 1 Put on a gown when entering the room 2 Place the client with another client who has TB 3 Wear a particulate respirator when caring for the client 4 Don a surgical mask with a face shield when entering the room

3 A high-particulate filtration mask that meets Centers for Disease Control (CDC) performance criteria (Canada: Public Health Agency of Canada [2013] Canadian Tuberculosis Standards, 7th edition) for a tuberculosis respirator must be worn to protect healthcare providers from exposure to the Mycobacterium tuberculosis organism. Airborne transmission-based precautions do not require a gown unless contact with respiratory secretions is anticipated. The client should be placed in a private room with negative pressure and multiple full air exchanges per hour vented to the outside environment. A surgical mask with a face shield is inadequate to prevent transmission of the tuberculosis microorganism.

While obtaining the client's health history, which factor does the nurse identify that predisposes the client to type 2 diabetes? 1 Having diabetes insipidus 2 Eating low-cholesterol foods 3 Being 20 pounds (9 kilograms) overweight 4 Drinking a daily alcoholic beverage

3 Excessive body weight is a known predisposing factor to type 2 diabetes; the exact relationship is unknown. Diabetes insipidus is caused by too little antidiuretic hormone (ADH) and has no relationship to type 2 diabetes. High-cholesterol diets and atherosclerotic heart disease are associated with type 2 diabetes. Alcohol intake is not known to predispose a person to type 2 diabetes.

The nurse is teaching a client to care for her episiotomy after discharge. Which priority instruction should the nurse include in her instructions? 1 Rest with legs elevated at least two times a day. 2 Avoid stair climbing for several days after discharge. 3 Perform perineal care after toileting until healing occurs. 4 Continue sitz baths three times a day if they provide comfort

3 Prevention of infection is the priority. Resting should be encouraged; however, it is not the priority at this time. Stair climbing may cause some discomfort but is not detrimental to healing. There is no limit to the number of sitz baths per day that the client may take if they provide comfort.

A client asks the nurse at the prenatal clinic whether she may continue to have sexual relations while pregnant. What is one indication that the client should refrain from intercourse during pregnancy? 1 Fetal tachycardia 2 Presence of leukorrhea 3 Premature rupture of membranes 4 Imminence of the estimated date of birth

3 Ruptured membranes leave the products of conception exposed to bacterial invasion. Intact membranes act as a barrier against organisms that may cause an intrauterine infection. Fetal tachycardia may occur during sex, but there is no evidence that it is harmful for the fetus. Leukorrhea is common because of increased production of mucus containing exfoliated vaginal epithelial cells; intercourse is not contraindicated by leukorrhea. Intercourse is not contraindicated near the estimated date of birth if the membranes are intact; modification of sexual positions may be needed because of the enlarged abdomen.

A nurse instructs a client with viral hepatitis about the type of diet that should be ingested. Which lunch selected by the client indicates understanding about dietary principles associated with this diagnosis? 1 Turkey salad, french fries, sherbet 2 Cottage cheese, mixed fruit salad, milkshake 3 Salad, sliced chicken sandwich, gelatin dessert 4 Cheeseburger, tortilla chips, chocolate pudding

3 The diet should be high in carbohydrates, with moderate protein and fat content. A salad, chicken and gelatin meal is the best choice. Turkey salad, french fries, and sherbet are too high in fat. Cottage cheese, mixed fruit salad, and a milkshake are dairy products and may cause lactose intolerance; the hepatitis virus injures the intestinal mucosa and reduces the client's ability to metabolize lactose. Cheeseburger, tortilla chips, and chocolate pudding are too high in fat.

While mopping the kitchen floor, a client at 37 weeks' gestation experiences a sudden sharp pain in her abdomen with a period of fetal hyperactivity. When the client arrives at the prenatal clinic, the nurse examines her and detects fundal tenderness and a small amount of dark-red bleeding. What does the nurse conclude is the probable cause of these clinical manifestations? 1 True labor 2 Placenta previa 3 Partial abruptio placentae 4 Abdominal muscular injury

3 Typical manifestations of abruptio placentae are sudden sharp localized pain and small amounts of dark-red bleeding caused by some degree of placental separation. True labor begins with regular contractions, not sharp localized pain. There is no pain with placenta previa, just the presence of bright-red bleeding. There are no data to indicate that the client sustained an injury.

A nurse is caring for a client who is positive for hepatitis A. Which precautions should the nurse take? 1 Wear a gown when entering the client's room. 2 Use caution when bringing in the client's food. 3 Use gloves when removing the client's bedpan. 4 Wear a protective mask when entering the client's room.

3 Use gloves when removing the client's bedpan. The virus is present in the stool of clients with hepatitis A; therefore, standard precautions should be followed when handling excretions. The virus also may be present in urine and nasotracheal secretions. The Centers for Disease Control and Prevention (CDC) (Canada: Public Health Agency of Canada (PHAC)) indicate that only standard precautions are necessary when caring for a client who is positive for the presence of hepatitis A; if a client is incontinent or using an incontinence device, the CDC (Canada: PHAC) recommends that contact precautions be implemented. Bringing food to a client requires no precautions; however, disposable utensils should be used and utensils discarded following standard precautions because the client's nasotracheal secretions contain the virus. Hepatitis A usually is not transmitted via the air.

An adolescent woman who has become sexually active asks the nurse, "What's the most effective way to prevent a pregnancy?" Which method of preventing pregnancy should the nurse tell her is most effective? 1 Birth control pills 2 Spermicidal foam 3 Condoms 4 An intrauterine device

4 According to the U.S. Centers for Disease Control and Prevention, having an intrauterine device inserted provides a 99% effective means of preventing pregnancy. The oral contraception pill has a high 91% effective rate when used correctly. Condoms are 82% effective and the barrier can reduce (but not eliminate) the risk of sexually transmitted infections. Spermicidal foam is a 72% effective means of preventing pregnancy; however, its effectiveness also depends on correct, consistent use. Although refraining from sexual intercourse is the most effective form of birth control (100% effective), this client has come to the nurse for advice about how to prevent pregnancy while being sexually active.

The nurse is caring for an assignment of postpartum clients. Which factor puts a client at increased risk for postpartum hemorrhage? 1 Breastfeeding in the birthing room 2 Receiving a pudendal block for the birth 3 Having a third stage of labor that lasts 10 minutes 4 Giving birth to a baby weighing 9 lb 8 oz (4309 g)

4 The chance of postpartum hemorrhage is five times greater with large infants because uterine contractions may be impaired after the birth. Early breastfeeding will stimulate uterine contractions and lessen the chance of hemorrhage. Having a pudendal block for the birth does not contribute to postpartum hemorrhage, because the anesthetic for a pudendal block does not affect uterine contractions. Ten minutes is a short third stage; a prolonged third stage of labor, 30 minutes or more, may lead to postpartum hemorrhage.

A male infant is born at 28 weeks' gestation weighing 2 lb 12 oz (1247 g). What does the nurse expect to note when performing an assessment? 1 Staring eyes 2 Absence of lanugo 3 Descended testicles 4 Transparent red skin

4 Transparent red skin is expected due to the absence of subcutaneous fat tissue. Preterm infants born nearer to term have open, staring eyes. Preterm infants are generally born with large amounts of lanugo, which begins to thin just before term and by 40 weeks is found only on the shoulders, back, and upper arms. The preterm infant's scrotum is small, and the testicles are usually high in the inguinal canal.

Alpha-fetoprotein (AFP)

A blood test that measures the level of alpha-fetoprotein in the mothers' blood during pregnancy as an indicator of possible birth defects in a fetus

The nurse is caring for a client with a hiatal hernia. The client states that favorite beverages include ginger ale, apple juice, orange juice, and cola beverages. Of the four the client listed, which is the only beverage that should remain in the client's diet? 1 Ginger ale 2 Apple juice 3 Orange juice 4 Cola beverages

Correct 2 Apple juice Apple juice is not irritating to the gastric mucosa. Carbonated beverages like ginger ale distend the stomach and promote regurgitation. The acidity of orange juice aggravates the disorder. Most colas should be avoided because they contain caffeine, which causes increased acidity and aggravates the disorder; also they are carbonated, which distends the stomach and promotes regurgitation.

Nägele rule for EBD

Determine first day of last menstrual period (LMP), subtract 3 months, add 7 days plus 1 year • Alternatively add 7 days to LMP and count forward 9 months

Cholecystokinin

a hormone that is secreted by cells in the duodenum and stimulates the release of bile into the intestine and the secretion of enzymes by the pancreas.

Methicillin-resistant Staphylococcus aureus (MRSA)

a strain of the bacterium Staphylococcus aureus that has become resistant to the antibacterial action of the antibiotic methicillin, a form of penicillin

multiparous

a woman who has given birth two or more times

tubal ligation

blocking the fallopian tubes to prevent fertilization from occurring

Nonstress test (NST)

for antepartum evaluation of fetal well being performed during third trimester. noninvasive test that monitors the fhr to fetal movement. a doppler transducer(used to monitor the fhr) and a tocotransducer(used to monitor uterine contractions) are attached externally to a client's abdomen to obtain tracing strips. client pushes a button when she feels the fetus move.

Thalassemia

inherited defect in ability to produce hemoglobin, leading to hypochromia

positive rebound tenderness

is a clinical sign that is elicited during physical examination of a patient's abdomen by a doctor or other health care provider. It is indicative of peritonitis. It refers to pain upon removal of pressure rather than application of pressure to the abdomen.

alpha-fetoprotein test

measures the level of AFP in pregnant women during the second trimester of pregnancy. Too much or too little AFP in a mother's blood may be sign of a birth defect or other condition.

Addison's disease

occurs when the adrenal glands do not produce enough of the hormones cortisol or aldosterone

dysmenorrhea

painful menstruation

episiotomy

surgical incision of the perineum to enlarge the vagina and so facilitate delivery during childbirth

carotid sinus massage

the physician massages over one carotid artery for a few seconds, observing for a change in cardiac rhythm.


Ensembles d'études connexes

Introduction to Neuroscience, Part 3

View Set

Unit 5: States of Consciousness: 5.2 Hypnosis

View Set